Sie sind auf Seite 1von 136

Anglo-Chinese Junior College

H2 Mathematics 9740
2009 JC 2 PRELIM Solution

Paper 1:
1

Solving for point of intersection:
2
5 2 4 20
2 2 5 0 1 6
2
x x x x
x
+
= = = =
For
5
2 x
x
> , 1 6 0 or 1 6 x x s < > +

Using previous results,
For
5
2 x
x
> , 1 6 0 (rej.) or 1 6 x x < < > +
( )
1 6 or 1 6 x x < + > +

2
( ) ( )
( ) ( )
( )
( )
1
1
1 1
1
1 1
1
2
2 2
1
1
( 1)
1
1
1
1
+
+
= =
+
+
=
+
+
+
=
=

= + =

n n
r
r r
r r
n
r
n
r
n
n
n
n
n
U U e e
U U e e
e e
U e e e
e
U e



( )( )
3 4 5 1
2 3 4
1
1 4
2
...
n
n
n n
U U U U e e e e
e
+
+
=
=


3 Let x, y and z be the cost of the flights for each parent, each
grandparent and each child respectively.
2 2 1959 ----- (1)
2 3 2196 ----- (2)
2 4 2640 ----- (3)
2 ?
x y z
x y z
x y z
x y z
+ + =
+ + =
+ + =
+ + =

Solving (1), (2) & (3) using GC,
x = 453, y = 354 and z = 345
Cost for Tan family = $[453 + 354 + 2(345)] = $1497


4 (i)
1
1
2
S = ,
2
2
3
S = ,
3
3
4
S = ,
4
4
5
S =


4 (ii) ( )
1
f 1
2
= , ( )
1
f 2
3
= , ( )
1
f 3
4
= , ( )
1
f 4
5
=
1
f ( )
1
n
n
=
+



4 (iii) Let P
n
denote the statement
1
1
1
n
S
n
=
+
, n
+
e .



For n = 1, LHS =
1
1
2
S = RHS =
1 1
1
1 1 2
=
+

LHS = RHS.
P
1
is true.
Assume P
k
true for some k
+
e , i.e.
1
1
1
k
S
k
=
+

Prove that P
k+1
is true, i.e.
1
1
1
2
k
S
k
+
=
+

( )( )
( )( )
( )( )
1
1
LHS
1 2
1 1
1
1 1 2
1
1
1 2
1
1 RHS
2
k k
S S
k k
k k k
k
k k
k
+
= = +
+ +
= +
+ + +
+
=
+ +
= =
+

P
k+1
is true if P
k
is true. But P
1
is true.
By the principle of mathematical induction, P
n
is true
n
+
e .

5 (i)
( ) f y x ' =



5 (ii)
( )
1
f
y
x
=



5 (iii) ( )
2
f y x =

y
x
O
O
y
x



6 (i)

Since f is one-one for 0 1 x s s ,
1
f

exists.
( )
2
2
2 2
1 1 1 1 0 1
y x x
x x y x
= +
= + = s s

( )
1
f 1 1, 1 2 x x x

= s s

6 (ii)
Range of f = | |
1, 2 Domain of g = | | 0, 3
Since
f g
R D _ , gf exists.
( )
2
2
2 4
gf , 0 1
2 3
x x
x x
x x
+
= s s
+


6 (iii)
Restricted Domain of g = | |
1, 2
Range of gf =
4 3
,
3 2
(
(



7 (i)
Differentiating
1 1 1
x y a
+ = w.r.t. x ,
2 2
1 1
0
dy
x y dx
=



2
2
dy y
dx x
=

Since 0
dy
dx
< , y is a decreasing function.

7 (ii)
There are no stationary points on the curve because 0
dy
dx
= as
0 y =

7 (iii)
Equation of tangent is
( )
( )
2
2
2
2
1
2
2
a
y a
x a
a



y
x
O




f
O
1
1
2


Therefore, 4 y x a = +

Substituting 4 y x a = + into eqn of curve
1 1 1
4 x a x a
+ =



2 2
4 4 0 x ax a + =
( )
2
2 0 x a =
2 x a =
Since only one value of x is obtained,
the tangent does not meet the curve again.
8 (a)

Domain of
1
cos x

: 1 1 x s s
Range of
1
cos x

:
1
0 cos x

s s t
Domain of
1
tan x

: x < <
Range of
1
tan x

:
1
tan
2 2
x

t t
< <

8 (b)

1 1
sin sin
3 3
sin or sin
3 3
x x
x x

t t
= =
t t | | | |
= =
| |
\ . \ .

3

2
x =


8 (c) The graph of
x
y e

= can be obtained from the graph of


ln y x = by a reflection in the line y x = .




1
1



O 1 1


For ln
x
x e

> , 0 x < < o


9 (i)






1
du dy
u x y
dx dx
= + = +

d.e. becomes
2
2 0
du
u
dx
= after replacing
dy
dx
and x y +
Therefore,
2
2
du
u
dx
= +

9 (ii)

2
1
2
du dx
u
=
+
} }



1
1
tan
2 2
u
x C

| |
= +
|
\ .


( )
tan 2
2
u
x D = + where D = 2C
( )
2 tan 2 y x D x = +

When x = 0, y = 2 . Therefore, 2 2 tan D = .
1
tan 2 D

=


The particular solution is
( )
1
2 tan 2 tan 2 y x x

= +

9 (iii)
( ) ( )
1
2 tan 2 tan 2 A x x x

= +

( ) d xy
dA
dz dz
= =
dy dx
x y
dz dz
+

When x = 0, y = 2 therefore,
3
2
dx
dz
=

10 (a)

(2 ) 9 16 z i w i + + = + (1)
*
3 z w i + = (2)
Substitute
*
3 w i z = into equation (1)
*
(2 )(3 ) 9 16 z i i z i + + = +
*
( 2 ) ( 3 6 ) 9 16 z i z i i + + + = +
*
( 2 ) 6 10 z i z i + = +
Let z x iy = +
( ) ( 2 )( ) 6 10 x iy i x iy i + + = +
( ) ( 3 ) 6 10 x y i x y i + + = +





Equating real parts: 6 6 x y x y = + = (3)
Equating imaginary parts: 3 10 x y + = (4)
Solving equations (3) and (4): 2 x = and 4 y =
2 4 z i = +
3 (2 4 ) 2 7 w i i i = = +
10 (b)
3
z i =
3
exp{ (2 )}
2
z i n
t
t = + where n Z e
2
exp{ ( )}
3 6
n
z i
t t
= +
1, n = exp{ ( )}
2
z i
t
=
0, n = exp{ ( )}
6
z i
t
=
1, n =
5
exp{ ( )}
6
z i
t
=
3
w i =
( ) ( )
*
*
3
w i =
( )
3
*
w i =
From the results of (a) above,
*
exp{ ( )}
2
w i
t
=
*
exp{ ( )}
6
w i
t
=
*
5
exp{ ( )}
6
w i
t
=
Hence exp{ }
2
w i
t
= , exp{ ( )}
6
i
t
,
5
exp{ ( )}
6
i
t


11 (i)













cot 2
4
y x
t | |
= +
|
\ .

( )
2 2
cos 2 .2 2 1
4
dy
ec x y
dx
t | |
= + = +
|
\ .

2
2
4
d y dy
y
dx dx
=
2
3 2
3 2
4
d y d y dy
y
dx dx dx
(
| |
= +
(
|
\ .
(

. Therefore, 4 k =

11 (ii)
When x = 0, y = 1, 4
dy
dx
= ,
2
2
16
d y
dx
= ,
3
3
128
d y
dx
=


Therefore,
2 3
64
cot 2 1 4 8
4 3
x x x x
t | |
+ ~ +
|
\ .


11 (iii) Differentiating the expansion above,
2 2
2cos 2 4 16 64
4
ec x x x
t | |
+ ~ +
|
\ .


Therefore,
2 2
cos 2 2 8 32
4
ec x x x
t | |
+ ~ +
|
\ .

Let
13
2
4 50
x
t t
+ = . Then
200
x
t
=

Hence,
2
2
13
cos 2
50 25 1250
ec
t t t | |
= +
|
\ .
where
1 1
2, ,
25 1250
a b c = = =


12 (i) Let N be the foot of perpendicular from A to the plane
1
p .
1
1
1
AN
| |
|
=
|
|
\ .

1 1 1
2 1 2
4 1 4
ON OA AN

+ | | | | | |
| | |
= + = + = +
| | |
| | |
+
\ . \ . \ .

Since N is a point on the plane
1
p ,
1
. 1 7
1
ON
| |
|
=
|
|
\ .

1 1
2 . 1 7
4 1

+ | | | |
| |
+ =
| |
| |
+
\ . \ .

( 1 ) (2 ) (4 ) 7 + + + + + =
2 =
2
1
3
1 1
2 1
2 2 8
3 3
4 14
2
4
3
ON

| |
+
|
+ | | | | |
| |
|
= + = + =
| |
|
| |
+ |
\ . \ .
|
+
|
\ .



12 (ii)
4 1
3 . 1 4 3 0 7
0 1
| | | |
| |
= + + =
| |
| |
\ . \ .
(satisfies equation of plane
1
p )
4 1
3 . 1 4 3 0 1
0 a
| | | |
| |
= + =
| |
| |
\ . \ .
(satisfies equation of plane
2
p )
Hence the point (4, 3, 0) is on both planes
1
p and
2
p .
(4, 3, 0) is then a point on the line where the 2 planes

intersect.

1 1 1
1 1 1
1 2
a
a
a
+ | | | | | |
| | |
=
| | |
| | |

\ . \ . \ .
is the direction vector of the line ,
Hence r
4 1
3 1
0 2
a
a
+ | | | |
| |
= +
| |
| |

\ . \ .
is the equation of line .
12 (iii)
If the planes
3
p with equation r
1
. 2
3
b
| |
|
=
|
|
\ .
intersects with
1
p and
2
p at line , then
1 1
2 . 1 0
3 2
a
a
+ | | | |
| |
=
| |
| |

\ . \ .

(1 ) 2(1 ) 6 0 a a + + =
3 a =
The line lies in the plane
3
p and thus
4
3
0
| |
|
|
|
\ .
lies in the plane
3
p
also.
4 1
3 . 2 4 6 0 10
0 3
b b
| | | |
| |
= = + + =
| |
| |
\ . \ .



12 (iv)
0 1 1
2 2 4
5 4 1
AB OB OA
| | | | | |
| | |
= = =
| | |
| | |
\ . \ . \ .
.
Given the length of projection of line segment AB on the line
is
1
6
.
1
. 1
2
1
1 6
1
2
a
AB a
a
a
+ | |
|

|
|

\ .
=
+ | |
|

|
|

\ .


2 2 2
(1 ) 4(1 ) 2
1
6
(1 ) (1 ) ( 2)
a a
a a
+
=
+ + +

2
5( 1) 1
3
( 3)
a
a

=
+


2
2
25( 1) 1
( 3) 3
a
a

=
+

2 2
75( 1) 3 a a = +
2
74 150 72 0 a a + =
150 1188 75 247
2(74) 74
a

= =
0.781 a = or 1.25 a =


Anglo-Chinese Junior College
H2 Mathematics 9740
2009 JC 2 PRELIM Marking Scheme
Paper 2:
1
Equation of line AB: r
1 3
2 4
1 1

| | | |
| |
= +
| |
| |
\ . \ .
or r
4 3
2 4
2 1

| | | |
| |
= +
| |
| |
\ . \ .


C is on the line AB:
1 3
2 4
1
OC

+ | |
|
=
|
|
+
\ .

Let AOC BOC u = =
. .
cos
OAOC OBOC
OA OC OB OC
u = =

Hence,
. . OAOC OBOC
OA OB
=

1 1 3 4 1 3
2 . 2 4 2 . 2 4
1 1 2 1
1 1
2 2 2
1 1



+ + | | | | | | | |
| | | |

| | | |
| | | |
+ +
\ . \ . \ . \ .
=
| | | |
| |
| |
| |
\ . \ .


| |
2 2(1 3 ) (2 4 ) (1 )
(1 3 ) 2(2 4 ) (1 )
6 2 6


+ + +
+ + + +
=
(3 8 1) (1 4 1) (6 4 1) (2 2 1) + + + + = + + + +
15 5 =
1
3
=
3
2
1
3
2
OC
| |
|
=
|
|
\ .

3
1
4
3
1
AC OC OA
| |
|
= =
|
|
\ .

3
2
4
3
1
CB OB OC
| |
|
= =
|
|
\ .

1
2
AC
CB
=
: 1: 2 AC CB =


2 (i)
( )
2
2
2
2 2 1
2
( )( 1)
2 1 ( )( ) ( ) ...
2 2! 2
( 1)
2 1 ...
2 8
b
b
b
b
b
ax
ax
ax b b ax
b
ab a b b
x x

| |
=
|
\ .
| |
= + + +
|
\ .
| | +
= + + +
|
\ .

Comparing constant term:
1
2 3
8
b
b

= =
Comparing coeff of x:
1 9
3
8 2 16
ab
a
| |
= =
|
\ .

Comparing coeff of x
2
:
2
1 ( 1) 27

8 8 16
a b b
c
| | +
= =
|
\ .



2 (ii)
2
( )( 1)...( 1)
coefficient of 2 ( )
! 2
1 ( 3)( 4)...( 2) 3
( )
8 ! 2
1 (3)(4)...( 2) 3
( 1) ( )
8 ! 2
1 ( 1)( 2) 3
( )
8 2 2
1 3
( 1)( 2)( )
16 2
r b r
r
r r
r
r
b b b r a
x
r
r
r
r
r
r r
r r

+
=

=
+
=
+ +
=
= + +

1 3
,
16 2
p q = =

3 (i)











3 (ii)





Required area =
5
2
y dx
}

= ( )
5
2
ln 2 t t dt
}

= 2
5
2 2
5
2
2
1
ln .
2 2
t t
t dt
t

(

`
(


)
}

=
5 3
ln5 ln 2
2 2


Therefore,
5 3
, 1,
2 2
o | = = =



2 5

3
(iii)
Required volume =
( )
2 2
2
5
2
1 1
ln5 5 ln 2 2 ln
2 2
x dx t t t
| | | |

| |
\ . \ .
}






= 5.75 square units

4 (a)
2 2
1 1 1 2 z i = + = + = ,
( )
2
2
3 3 1 2 w i = + = + =
arg( ) arg(1 )
4
z i
t
= + = , arg( ) arg( 3 )
6
w i
t
= + =
2
2
z
z
w w
= =
arg arg( ) arg( )
z
z w
w
| |
=
|
\ .

arg
4 6 12
z
w
t t t | |
= =
|
\ .

( ) ( )
1 1 3 1
3 1 3 1
4 3 3 3
z i i i
i
w i i i
+ +
(
= = = + +

+ +


2
cos sin
2 12 12
z
i
w
t t | |
= +
|
\ .
( ) ( )
1
3 1 3 1
4
i
(
= + +


Equating real parts:
2 1
cos ( 3 1)
2 12 4
t
= +
Equating imaginary parts:
2 1
sin ( 3 1)
2 12 4
t
=
3 1 3 1
tan 2 3
12 3 1 3 1
t
= =
+



3 1 3 1
3 1 3 1

=
+

2 3 =

4 (b)



-1
y
1
0
x
2
A(2, 1)
P
Q



AQ = 2

2
cos 2 2
4 2
AN AQ
t
| |
= = =
|
|
\ .

2
sin 2 2
4 2
NQ AQ
t
| |
= = =
|
|
\ .

x-coordinate of Q: 2 2 +
y-coordinate of Q: 1 2 +
x-coordinate of P: 2 2
y-coordinate of P: 1 2
Coordinates of Q: ( 2 2 + , 1 2 + )
Coordinates of P: ( 2 2 ,1 2 )
Hence (2 2) (1 2) z i = + + +
or (2 2) (1 2) z i = +
5 The number of seating arrangements = 4! = 24

Number of ways = 3! 2! = 12




Number of ways = 4! 2! 6 = 288

6 (i) Let X be the random variable for the number of vehicles arriving in 2
minutes
X ~ Po(20)
P(X = 18) = 0.0844

6 (ii) Vehicles: 180 min = 3hr 1800

Variance = Var(0.7X) = 0.7 x 0.7 x 1800 = 882

6 (iii) Let C be the random variable for the number of cars
C ~ B(50, 0.7)
Since np = 50 x 0.7 = 35 > 5 and nq = 50 x 0.3 = 15 > 5
Using Normal approximation to Binomial
C ~ N(35, 35(0.3) i.e N( 35, 10.5)
P(C>30) = P( C > 29.5) (with cc)
= 0.955

7 (i) Stratified sampling is more representative of the entire population.
Random sampling cant ensure that both genders are represented
appropriately .
(A random sample might produce all boys and their eating
preferences may be different from girls)

7 (ii) Biased or not random. The population of leaves beyond the students
reach has been excluded .



8 Let W be the random variable for the number of matches of tennis
won by Wayne
X ~ B(120,
2
1
)
P(W > 70) = 0.0412
Probability that Wayne won 70 or more matches is only 0.0412.

Q
A
N
It does not seem likely that Henry and Wayne are equally skilled
players.
9 (i) n = 50, large using Central Limit theorem
_
X ~ N(20 000, )
50
2200
2


P( 000 20
_
X < 800) = P(19200
_
X 20 800) = 0.990

9 (ii) Let Y be the random variable for number of samples with sample
mean within $800 of the population mean
Y ~ B(5, 0.989868)
P(Y= 3) = 9.96 x 10
-4


9 (c) Assume :
P(success ) = 0.989868 is a constant for each trial

10 (i) Let X be the random variable for the length of a pickle
X ~ N(9 , 0.8
2
)
(a) P(X < 8 ) = 0.106
(b) P(X > 10. 5) = 0.0304

10 (ii) Since 10.6% of the pickles are discarded because they are too short ,
it is impossible to reduce the total rejects to 5%

10 (iii) Let L be the random variable for the number of pickles which are too
long
L ~ B(100, 0.0304)
P(L >8) = 1 P(L 8 s )
Since n = 100 large, p = 0.0304 < 0.1 and np = 3.04 < 5
Using Poisson approximation to Binomial
L ~ Po(3.04)


P( L> 8) = 0.00413

10 (iv) P( both pickles to meet his guaranteed standard | X
1
+ X
2
< 16 cm)
=
1 2
1 2
( ' & 16)
( 16)
P both pickles fail his g teed standard X X
P X X
+ <
+ <

=
) 16 (
)) 8 ( (
2 1
2
< +
<
X X P
X P
.(***)
=
03854989 . 0
) 1056498 . 0 (
2
= 0.290

11 (i)


11(ii)
To test H
0
: = 3.6
against H
1
: < 3.6 at 1 % level of significance
Under H
0
,
_
X ~ N(3.6,
n
2
2 . 1
)
Test statistics: Z =
n
X
o

0
_

=
n
2 . 1
6 . 3 5 . 3
=
n
2 . 1
1 . 0

p-value = P(
_
X < 3.5) = P( Z <
n
2 . 1
1 . 0
) < 0.01= P( Z < -2.32635) ,
Reject H
0,
hence
n
2 . 1
1 . 0
< -2.32635.

n > (2.32635 x 12)
2
= 779.314
Hence minimum n = 780
11 (iii) Carry out same test as above at 5% level of significance
n = 15 ,

x = 34 Hence 13 . 3
15
47
_
= =

=
n
x
x

= 1 ) 1 ( x x = 47 1(15) = 32
s
2
=

]
)) 1 ( (
) 1 ( [
1
1
2
2
n
x
x
n
=
14
1
[90 - ]
15
2
) 32 (
= 1.55238
using the t-distribution
Test statistics t =
n
s
X
0
_

=
15
55238 . 1
6 . 3 1333 . 3
= - 1.45
p-value = 0.0845 > 0.05
Do not reject at 5% level of significance and conclude that there is
insufficient evidence that the modification has decreased the mean
diameter of the ball bearings at 5% level of significance.

11 (iii) If = 3.6 , in fact 100p% of tests using the same method will lead to
wrongly rejecting = 3.6 or
p value is the probability of wrongly rejecting the hypothesis that =
3.6

12 (i) r = -0.952


For the depths sampled the moisture content decreases
approximately linearly with an increase in the depth of the sample.











Since r is close to 1 , this implies that the regression line of m on x and
x on m are almost identical or close to each other


12 (ii) m = - 2.2048x + 83.583.
When m = 50 , x =
2048 . 2
50 583 . 83
= 15.232 = 15.2
Reliable since the value of m is within the range of the given data. Not
extrapolating.

12 (iii)
2
yx
r = (-0.904)
2
= bd = -2.33 d

Hence d = -0.351
Hence regression line of x on y is x = -0.351y + C
Since 5 . 17
_
= x and 225 . 9 50 ) 5 . 17 ( 33 . 2
_
= + = y

(17.5, 9.225) also falls on the line of regression of x on y
Hence C = 20.7
Regression line of x on y is x = -0.351y + 20.7
13 Let U and R be the events that United and Rover won the match
respectively.
P(match is still undecided after 1 round )
= P( R U ) + P(R U) = 0.8 0.9 + 0.2 0.1 = 0.74 =
50
37


13 (i) P(United won in less than 3 rounds| United scores a goal in the first
round)
=
round first the in goal a scores United P
round first the in goal a scores United and rounds than less in won United P
(
) 3 (

=
8 . 0
1 . 0 8 . 0 9 . 0 8 . 0 1 . 0 8 . 0 + x x
..(*)
= 0.172

13 (ii) Let X be the random variable for the number of rounds played
P(match is decided in at most n rounds) > 0.98
P(X ) n s > 0.98
P(X = 1) + P(X = 2) + P(X = 3) + +P(X = n) > 0.98
Hence
50
13
+
50
37
(
50
13
)+
2
)
50
37
(
50
13
++(
1
)
50
37
n
50
13
> 0.98 (**)
50
13
(
(

|
.
|

\
|
+ +
|
.
|

\
|
+ +
1 2
50
37
........
50
37
50
37
1
n
> 0.98
n
|
.
|

\
|
50
37
< 0.02
Hence n >
50
37
ln
02 . 0 ln
= 12.992
Least n = 13




DUNMAN HIGH SCHOOL
H2 Mathematics
2009 Yr 6 Prelim Paper 1 Suggested Solns



1 Let x, y, z be the number of B
1
, B
2
, B
3
bears made in the month.
180 x y z + + =

5 4 3 650 x y z + + =

20 12 9 2100 x y z + + =


Augmented matrix M =
1 1 1 180
5 4 3 650
20 12 9 2100
| |
|
|
|
\ .

RREF(M) =
1 0 0 30
0 1 0 50
0 0 1 100
| |
|
|
|
\ .

Thus x = 30, y = 50, z = 100


2 From GC, a = 0
1
tan
tan
tan
x y
x y
x y

=
=
=

Since 0 x s and 0
2
y
t
s < , therefore tan x y =
1
f : tan x x

, 0
2
x
t
s <



3(a)
2 2
9 4 36 0 x y x =
( )
2
2
9 2 4 36 x y =
( )
2
2
2
1
4 9
x
y

= i.e Hyperbola
Asymptotes:
( )
( ) ( )
2
2
2
3 3
2 or 2
4 9 2 2
x
y
y x y x

= = =










(b)(i)
Asymptotes: and 1
2
x
y x = =
y
x
4
2
3
3
2
y x = +
( )
3
2
2
y x =
(ii)
( )
2
d 1
0
d 2
1
y A
x
x
= =


( )
2
1 2 x A =
Therefore, for C not to have stationary points, A < 0.


4(a)
( ) ( )
g( ) 2 g( ) g( )
f ( ) tan e f '( ) sec e g'( )e
x x x
x x x = =

2
f '(5) sec (e)( 3)e 9.81 = =
(b)
2 2
2
d (2 1)(2 ) (2)
2 1 d (2 1)
x y x x x
y
x x x

= =



2
2
2 2
(2 1)
x x
x

=


Function is increasing
2
2
d 2 2
0 0
d (2 1)
y x x
x x

> >


2
2 ( 1)
0 0 or 1
(2 1)
x x
x x
x

> < >





5(a)
By ratio theorem,

2 5
3 2 1
4
4 2
1
3
3
0
OP
| | | |
| |
+
| | | |
| |
|
\ . \ .
= =
|
|
\ .

(b)
11 3 8
4 4
14 6 8
AB EC OE
| | | | | |
| | |
= = = +
| | |
| | |
\ . \ . \ .

(c)
2
1
1
AD
| |
|
=
|
|
\ .

Angle required =
1
3 2
4 1
6 1
cos 102
61 6

| | | |
| |
-
| |
| |
\ . \ .
=
(d)
11 5 6 3
1 1 4
14 2 12 6
BC k AB BC k
| | | | | | | |
| | | |
= = = + =
| | | |
| | | |
\ . \ . \ . \ .

2 & 8 1 9 k = = =




6(i) d
, where and arepositiveconstants.
d
d 2 3
Set ( ) 0, .
d 3 2
d 3
d 2
3
(1 )
2
(2 3 ) (2 3 )
2
m
R Bm R B
t
m
R B B R
t
m
R Rm
t
R m
R
m k m
=
= = =
=
=
= =

(ii)
3 3
3
d
(2 3 )
d
1 d
(2 3 ) d
1
ln | 2 3 |
3
ln | 2 3 | 3 3
d 3
(2 3 ) e ( (2 3 ) 0)
d 2 2
2
e
3
kt c
kt
m
k m
t
m
k
m t
m kt c
m kt c
m R
m R Rm m
t
m A

=
=

= +
=
+ = = = >
=

(iii) Any positive value of k
But A must be a positive value STRICTLY LESS than
2
3

e.g. k=1, A=
1
3
, then
3
1
[2 ]
3
t
m e

=
m

2/3


2/3-A

0 t













7
( )( )
4 2 3 1
1 2 1 2
r
r r r r r r
+
= +
+ + + +

(a)
1
1 1
2 3 1
1 2
2 3(1 ) 2 1
2
r r r
r
r r


+
+ +
| | | |
= + + +
| |
\ . \ .

Coefficient of r
3
=
47
16

(b)
( )( )
{
1 1
4 2 3 1
1 2 ( 1) ( 2)
2 3 1

(1) (2) (3)
2 3 1

(2) (3) (4)
2 3 1

(3) (4) (5)

2

(
n n
r r
r
r r r r r r
n
= =
| | +
= +
|
+ + + +
\ .
= +
+ +
+ +
+


}
3 1

2) ( 1) ( )
2 3 1

( 1) ( ) ( 1)
2 3 1

( 1) ( 2)
n n
n n n
n n n
+

+ +
+
+ +
+ +

( )( )
3 1 3 1
2 ( 1) ( 1) ( 2)
3 2 1
2 ( 1) ( 2)
3 2 4 ( 1) 3 3

2 ( 1)( 2) 2 1 2
n n n
n n
n n n
n n n n
= + +
+ + +
= +
+ +
( + + +
= =
(
+ + + +


(shown)
( )( )
1
4 3
1 2 2
n
r
r
r r r
=
+
<
+ +


( )( )
( )
( )
( )( )
3
1 1
3
1 1
4 4
( 2) 2 2
2
4 4 3
1 2 2
2
n n
r r
n n
r r
r r
r r r
r
r r
r r r
r
= =
= =
+ +
=
+ + +
+
+ +
< <
+ +
+








8(a)
( ) ( )
2
2 2
1 2
3 3
2 2
0 1
2 1 2
0 0 1
4 3 d 4 3 d 4 3 d
2 3 2 3
3 3
1 8 1
2 3 8 6 2 3
3 3 3

x x x x x x x x x
x x
x x x x
+ = + +
( (
= + +
( (

( ( (
= + + + +
( ( (

=
} } }
2

(b)
1 1
2
1
2
1 2
1
tan d tan d
1
1 2
tan d
2 1
1
tan ln(1 )
2
x x x x x x
x
x
x x x
x
x x x c

| |
=
|
+
\ .
=
+
= + +
} }
}

(c)
2
2
2 2
2
1
2
1 1
d sec d
tan sec
1
cos
d
sin
(sin )
1

x t t
t t
x x
t
t
t
t c
x
c
x

=
+
=
= +
+
= +
} }
}




9(a)
Common ratio =
q
p

( )
2
n n
n
S S S
S S

=
=

1
2
1 1
2 1 1
2 1
2 (shown)
n
n
n
n n
q
p
p
p
q q
p p
q
p
q
p
p q
| |
| |
|
|
|
\ .
\ .
=

| |
| |
| =
|
|
\ .
\ .
| |
=
|
\ .
=

(b)(i) First term of each row: 2, 4, 8, 14.
Difference between the first terms: 2, 4, 6. is AP with a = 2, d
= 2.
Sum of the first (n 1) differences
= ( ) ( )
2
1
2 2 2 2
2
n
n n n

+ = (



First term of nth row = Sum of the first (n 1) differences + 1
st

term =
2
2 n n +
(ii) Total no. of terms from 1
st
row to (n 1)th row = 1 + 2 + 3 +
(n 1)

( )
( )
( ) 1 1
1 1
2 2
n n n
n

= + = (



Sum of all terms from 1
st
row to (n 1)th row
( )
( )
2
1
2
2 2 2
2
n n
n n

(
= + +


( )( )
2
1 2
4
n n n n +
=


10(i)
At point A,

( ) ( ) ( ) 7 1 2 8 8 1 RHS + = =


(ii)
Normal vector of t
2
=
1 0 14
4 2 4
1 4 2
| | | | | |
| | |
=
| | |
| | |
\ . \ . \ .

Let u be the angle required.
14 7
4 2
2 1
108
sin 1 90 or
2 216 54 11664
t
u u
| | | |
| |
-
| |
| |
\ . \ .
= = = =

(iii)
Normal vector of t
1
=
7
2
1
| |
|

|
|
\ .
is parallel to direction vector of l
Since l is both perpendicular to t
1
and t
2
, t
1
and t
2
must be
parallel.

Alternative:
Normal vector of t
1
=
7
2
1
| |
|

|
|
\ .
=
14
1
4
2
2
| |
|

|
|
\ .

Since the normal vectors of t
1
and t
2
are parallel, t
1
and t
2
are
also parallel.
(iv) Observe that t
2
contains origin,
Shortest distance required =
( )
2
2 2
1 1
54
7 2 1

=
+ +



Alternative:
( ) Let 0,1, 2 B

0 1 1
1 8 7
2 8 6
AB
| | | | | |
| | |
= =
| | |
| | |

\ . \ . \ .

Shortest distance required =
( )
2
2 2
1 7
7 2
6 1
1
54
7 2 1
| | | |
| |
-
| |
| |

\ . \ .
=
+ +




(v) Let M be the foot of perpendicular of P on t
1
.
( )
3 7 7
7 2 2
7
8 1 1
1
2
2 54 54
1
PM PA
| | | | | | | |
| | | |
-
| | | |
| |
| | |
|
| \ . \ . \ .
= - = = |
|
|
|
\ .
|
|
|
\ .
n n

' '
7 4 3
1
2 2 1 3
2
1 0 1
OP PP OP = +
| | | | | |
| | | | |
= + =
|
| | |
\ .
| | |

\ . \ . \ .


Alternative:
Let l
2
be the line passing through point P and parallel to normal
vector of t
1
.
2
4 7
: 1 2
0 1
l s
| | | |
| |
= +
| |
| |
\ . \ .
r

4 7 7
1 2 2 1
0 1 1
s
( | | | | | |
( | | |
+ - =
( | | |
| | |
(
\ . \ . \ .

1
28 49 2 4 1
2
s s s s + + + = =

Let M be the foot of perpendicular of P on t
1
and P be the
point of reflection required.
1
2
2 & '
1
2
a
OM OP b
c
| |
| |
|
|
| = =
|
|
|
| \ .
\ .

Using midpoint theorem:
4 1 1 0 1
, 2,
2 2 2 2 2
a b c + + + | |
= = =
|
\ .

3
Thus ' 3
1
OP
| |
|
=
|
|

\ .



11(a)
2
d
2 2
d
d
2
d
d d d 1
d d d
C
C r
r
A
A r r
r
C C A
A r r r
= =
= =
= =

When cm,
d d d 2
d d d
d 2
3
d 3
r k
C C A
t A t k
C
k
t
=
= =
= =

(b)
2
2 2
2
2
1 1 1
2 2 ( )(2 ) 800
2 2 2
1
2 3 800
8
5
6 800
4
5 24 3200
xy xy x x x
xy x x
xy x
x xy
(
| |
+ + + =
(
|
\ .
(

(
+ + =
(

+ =
+ =

( )
2
2 3
2
2 3
3
2
1
(2 )
8
1
2
4
3200 5 1
2
24 4
800 1

3 6
1
1600
6
V xy x x
x y x
x
x x
x
x x
x x
(
= +
(

= +
| |
= +
|
\ .
=
=


For max. V,
2
d 1
(1600 3 ) 0
d 6
23.094 (reject negative ans.)
V
x
x
x
= =
=

y = 3.0230
Thus the values of x and y are 23.1 and 3.02cm resply.
2
2
d 1
( 6 ) 0
d 6
V
x x
x
= = <

Thus volume is maximum.









12(a)
i i i
i
2 2 2
1 e e (e e )
o o o
o

+ = + =
i
2
2cos e , .
2
o
o
o e

( ) ( )
4 4
i i
4
4
i
i
2
2
4 4
i i
4 4 4 4
2 2
4 4 i 2 i 2
4
4
1 e 1 e
2cos e 2cos e
2 2
2 cos e 2 cos e
2 2
2 cos (e e )
2
16cos (2i sin 2 )
2
32i cos ( ) sin(2 ).
2
o o
o
o
o o
o o
o o
o o
o
o
o
o
o

| |

|
\ .

+ +
| |
| |
| |
= |
| |
|
\ .
\ .
\ .
=
=
=
=

(b)
1
arg i
2 4
z
t | |
=
|
\ .

1
arg i[ ]
2i 4
z
t | |
=
|
\ .

i
arg(i) arg
2 4
z
t | |
+ + =
|
\ .

i 3
: arg
2 4
L z
t | |
+ =
|
\ .











L

cos
1
4
| ( 1) |
2
1
min | i |
2 2
h
z h
t
=

+ = =

From the graph above, observe that in order for the two loci
1
arg i
2 4
z
t | |
=
|
\ .
and i z c + = to have 2 intersection points,

1 1 1 1
we have | ( 1) |
2 2 2 2 2 2
c c < < < <




DUNMAN HIGH SCHOOL
H2 Mathematics
2009 Yr 6 Prelim Paper 2 Suggested Solns



1(i)



























(ii)
(iii)

















2
3 2
3 25 0 z az bz + + + =
Since 3 4i + is a root and the coefficients of the equation are real,
( ) f 2 y x =
y
x
3
1
4,
2
| |

|
\ .
y
x
1
( ) 2,1
( ) f y ' x =
2
x
( )
1
f
y
x
=
y
2
1
1
( ) 2, 2
O
O
O
3 4i is also a root.
( ) ( )
2 2
3 4i 3 4i ( 3) 16 6 25 z z z z z + = + = + ( (



Thus
3 2 2
3 25 ( )( 6 25) z az bz pz q z z + + + = + +
Comparing coefficient of
3
z : p = 3
Comparing constant term: 25q = 25 1 q =
Thus 6 17 a q p = =
25 6 69 b p q = =

The other roots of the equation are 3 4i and
1
3
.













Let
2 3
arg( ) z z o =
Then
3 1
arg( ) z z o =
2 3 3 1
arg( ) arg( ) rad z z z z + =



















3(a)(i)
1
1
tan
tan
2
d e
e
d 1
x
x
y
y c
x x

= = +
+

When x = 0, y = 1
0
1 e 0 c c = + =

Im(z)
Re(z)
3
4


z
1
z
2
z
3
o
o
t o
Thus
1
tan
e
x
y

=
(ii)
( )
( )
( )
1
tan
2 2
2
2
2
2
2
2
2
d e
d 1 1
d
1
d
d d d
1 2
d d d
d d
1 (2 1) 0
d d
x
y y
x x x
y
x y
x
y y y
x x
x x x
y y
x x
x x

= =
+ +
+ =
+ + =
+ + =

(iii)
( )
( )
( )
2
2
2
3 2 2
2
3 2 2
3 2
2
3 2
d d
1 (2 1) 0
d d
d d d d
1 2 (2 1) 2 0
d d d d
d d d
1 (4 1) 2 0
d d d
y y
x x
x x
y y y y
x x x
x x x x
y y y
x x
x x x
+ + =
+ + + + =
+ + + =

When x = 0, y = 1 (given)

2 3
2 3
d d d
1, 1, 1
d d d
y y y
x x x
= = =

Thus Maclaurin series is
2 3
1 1
1
2 6
y x x x = + + +

(iv)
1
tan
2
2
e d 1
1
d 2 1
x
y
x x
x x

= = + +
+

(b)(i)
Using GC, volume required =
( )
1
2
2
tan
0
e d 30.607 30.6
x
x

= =
}

(ii)
Using GC, volume required =
2
2
2 3
0
1 1
1 d
2 6
x x x x
| |
+ +
|
\ .
}

= 38.317 = 38.3
Percentage error =
38.317 30.607
100% 25.2%
30.607

=











4(i)
( )
1 1 1
1 1
3 2 5 5 2 ,
3 3
n n n n
u u u u u
+ +
+ = = =
n
n
u
n
a b +
1
1
3

( )
2
3
1+
2
13
9

( )
2
2
3
1+
3
19
27

( )
3
2
3
1+
Conjecture:

( )
2
3
1 ,
2
Thus
3
n
n
u n
a
+
= + e
=

(ii) Let P
n
be the statement

( )
2
3
1
n
n
u = + for all n
+
e .
When n = 1, LHS
1
1
(given)
3
u = =
RHS
( )
2
3
1
1
3
= + = = LHS
P
1
is true.

Assume P
k
is true for some k
+
e .
i.e.
( )
2
3
1
k
k
u = +
To prove P
k+1
is also true, i.e.
( )
1
2
1 3
1
k
k
u
+
+
= + .
LHS = ( )
1
1
5 2
3
k k
u u
+
=

1 2
5 2 1
3 3
k
(
| |
| |
( = + |
|
|
\ . (
\ .


1
1 2
3 2
3 3
2
1 RHS
3
k
k+
(
| |
= (
|
\ .
(

| |
= + =
|
\ .

Thus
1
P is true P is true
k k+
.
Since
1
P is true, and
1
P is true P is true
k k+
, by Mathematical
Induction, P
n
is true for all n
+
e .
(iii)
( )
2
3
1 1 1
1
1
1 as
diverges.
n n n
r
r
r r r
n
r
n
u
n
S
= = =
=
= +


(iv)
( )
( )
( )
( )
2
3
2
3
2
3
2
3
1
As , 0, 1
2
0.005
3
2 ln(0.005)
ln ln(0.005) 13.1
3 ln
Minimum integer 14
n
n
n
n
k
k
k
u
n u L
u L
k k
k
= +
=
| |
= = <
|
\ .
| |
< > =
|
\ .
=










































5(i)





Stratified Sampling

Category
Year Number of students No. of students in sample
1 340
340
1200
60 17 =
2 300 15
3 280 14
4 280 14
Choose 17, 15, 14 and 14 students from Year 1, 2, 3 and 4
respectively.

Randomness
Within each year group, assign a number to every student and
randomly select the required number of students.
(ii)
- The cafeteria can sell the preferred food of the respective
group of students during their designated breaks.
- The cafeteria can prepare the quantity of preferred food of
the respective group of students in the same proportion of
the student cohort of each group.































6(i) No of ways required = 4! 5! 2880 =
(ii)
No of ways required =
4
1
4! 4! 2 2 9216 C =









4
1
: choose one of the four possible slots for M and A C

4!: once M and A are fixed, all the girls are fixed

4!: arrange the remaining boys

2: M and A can permute

2: there are two sides of the table


G M A B G
B G B G B
X X X X X


































7(i) Required probability
=1 P( all seeds selected do not germinate)
16 15 14 13 12
1
20 19 18 17 16
232
323

| || || || || |
=
| | | | |
\ .\ .\ .\ .\ .
=

(ii) Let X be the number of seeds that germinate, out of 5.
X ~ B(5, 0.10)
Required probability P( 1) 1 P( 1) X X = > = s

= 1 0.91854
= 0.0815

Binomial distribution is used in part (ii) based on the
assumption that probability of success is a constant when the
sample is small as compared to a large population.





































8(i)
( ) P | G H =
1
4

(ii)
( ) P | H G =
1
2

(iii)
( ) P R E =
5
8


Since ( ) P 0 R E = ,
R & E are mutually exclusive.

( ) P | G H =
1
4
and ( )
2 1
P
8 4
G = =
Since ( ) ( ) P | P G H G = , G and H are independent.

Alternative
( ) P | H G =
1
2
and ( )
4 1
P
8 2
H = =
Since ( ) ( ) P | P H G H = , G and H are independent.

Alternative
( )
1
P P(card is blue and has number 5)
8
G H = =
( )
( )
( ) ( ) ( )
2 1
P
8 4
4 1
P
8 2
P = P P
G
H
G H G H
= =
= =


Thus G and H are independent.






















9(i)









(ii)
Outlier :

( ) 17,150

0.999 r =

3.5953 58.639 3.60 58.6 y x x = + = +

(iii)
When 17 x = , ( ) 3.5953 17 58.639 119.76 120 y = + = =

From GC, 0.999 r = or
2
1 r ~
Since the product moment correlation coefficient is almost +1, the
regression lines is almost an exact fit to the data. Thus the regression
lines Y on X and X on Y will almost coincide.







y
x
100
120
140
160
10 15 20 25




































10 Let X represent the weight loss with population mean .

From GC,
x =0.7
Unbiased estimate of population variance:
s
2
= (2.3118)
2
= 5.3444.

To test H
0
: 0 =
against H
1
: 0 >

One-tail test at 10% level (o = 0.10)

Since population variance is unknown and the sample size of 10
is small, assuming X is normally distributed and s
2
is a good
estimate of o
2
,
under H
0
, T =
0
2.3118/ 10
X
~ t (9).

From GC, p-value = P( 0.7 when 0) X > = = 0.18166 = 0.182.

Since p-value =0.182 > 0.10, we do not reject H
0
and conclude
that there is insufficient evidence at 10% level of significance
that the slimming companys claim is justified.
The probability of concluding that the pill helps people to lose
weight when it actually does not is 0.10.
p-value = 0.18166 2 = 0.36332 = 0.363
For H
0
to be rejected, p-value < o
Thus o > 0.363

























11(i) X~N(40, 4.5
2
), Y~N(20, 6.9
2
)
X + Y
1
+ Y
2
+ Y
3
~N(100, 163.08)
1 2 3
P( 120) 0.058658 0.0587 X Y Y Y + + + > = =
(ii) Let T = 20X 8Y
T~N(640, 11147.04)
P( ) 0.3 584.63 T k k < = =
(iii) Let W be the no. of observations of T that is more than k.
W ~ B(120, 0.7)
Since n = 120 is large, np = 84 > 5 and nq = 36 > 5,
W~ N(84, 25.2) approximately.
P(W >

100) = P(W > 99.5) after continuity correction
= 0.00101









































12(i) Let X be the no. of earthquakes in 10 years.
X ~ P
o
(0.0000236510) i.e. X ~ P
o
(0.073)
P(X > 2) = 1 P(X s 1) = 1 0.99746 = 0.00254
(ii) One earthquake in the year 2000 to 2005 and none in the year
2004 2009 One earthquake in the year 2000 to 2003 and
none in the year 2004 2009.
Let Y and W be the no. of earthquakes in 4 years and 6 years
respectively.
Y ~ P
o
(0.0292), W ~ P
o
(0.0438)

Required probability = P(Y = 1)P(W = 0)
= 0.028360 0.95715
= 0.027145 = 0.0271
(iii) Let V be the no. of periods where there are at least two
earthquakes, out of 200.
V ~ B(200, 0.00254)
V ~ P
o
(0.508) approximately.
P(V s q) > 0.999
Using the GC, q > 4
q P(V s q)
3 0.99815 < 0.999
4 0.99981 > 0.999

Since n =200 is large, np = 0.508 < 5.
Let T be the mid-day temperature of the city.
E(T) = 18, Var(T) = 16
Since n = 100 is large, by Central Limit Theorem,
16
~ N 18, approximately
100
T
| |
|
\ .

P(17 19) 0.988 T < < =


2009 HCI Preliminary Examinations H2 Mathematics Paper 1
Solutions

1.
Let
( 1)
be the proposition that for .
( 1)!
n n
a n n
P u n
n
+
+
= e


For n = 1, LHS of
1
P = 2a
RHS of
1
P =
(1 1)1 2
= 2
(1 1)! 1
a a
a
+
= =


LHS of
1
P = RHS of
1
P
Thus,
1
P is true.
Assume that
k
P is true for some k
+
e .
i.e.
( 1)

( 1)!
k
a k k
u
k
+
=


To prove that
1 k
P
+
is true:
1 2
2
2
( 2) ( 1)

( 1)!
( 2)( 1)
=
( 1)!
( 2)( 1)
=
!
k k
k
u u
k
k a k k
k k
a k k
k k
a k k
k
+
+
=
+ +
=

+ +

+ +

Thus
1 k
P
+
is true.
Since
1
P is true, and
k
P true
1 k
P
+
is true. By Mathematical
Induction, is true for all .
n
P n
+
e

Limit of sequence is 0.














2.
Given curve C is defined by:
2sin sin 2 , 2cos cos 2 ,
2 2

x t t y t t t = = s s
(i) The graph of C is









(ii)


Area of the shaded region
( )( )
( )
2
2
2
2
d 4 1
2cos cos 2 2cos 2cos 2 d 4
1.42478
1.4 correct to1d.p.

y x
t t t t t

=
=
=
=
}
}















x
y
O
0
2 2
1

1
2
y
O
0
2
x
3(i)
Concave downwards f ''( ) 0 x < so the gradient of f '( ) x <0
Strictly increasing f '( ) x >0
1 x > (ans)

(ii)
Stationary points: f '( ) x =0 3, 0 x =
x (3)

3 (3)
+
f '( ) x -ve 0 -ve




x 0

0 0
+
f '( ) x -ve 0 +ve


(iii)
























4 Complex No





x
y
1
3 1.5
x =1
Point of inflexion
at x = 3
Minimum point at
x = 0.







5
( )( )( ) ( )( ) ( )( )
1
g( )
3 2 3 1 3 4 3 2 3 1 3 1 3 4
1 (3 4) (3 2)
1
Solving, we get
6
a b
r
r r r r r r r
a r b r
a b
= =
+ + + + +
= +
= =


(i)
1
g( )
n
r
r
=

=
( )
f (1) f (2)
+f (2) f (3)
1 1
f (1) f ( 1)
................... 6 6
f ( ) f ( 1)
n
n n
| |
|

|
= +
|
|
+ +
\ .

1
g( )
n
r
r
=

=
( )( )
1 1 1
6 4 3 1 3 4 n n
| |

|
|
+ +
\ .


(ii)
( )( )( ) ( )( )( )
1 1
...
3 5 3 2 3 1 3 2 3 1 3 4 n n n n n n
+ +
+ + +

( )( ) ( )( )
2
1 1 1
g( ) g( ) g( )
1 1 1 1 1 1
6 4 6 4 3 5 3 2 6 3 5 3 2
n
r n r r
r r r
n n n n

= = =
| |
= =
|
\ .
| |
| |
= =
|
|
|

\ .
\ .




1
g( )
n
r
r
=

represents the sum of area of rectangles drawn with breadth


of unit 1 and height g(r) for r = 1, 2, 3, ., n as shown in diagram
while ( )
1
1
g d
n
x x
+
}
represents the area under the curve bounded by
r = 1 to r = n + 1 which is smaller than the sum of area of rectangles.



1 2 3 4 n n + 1
y = g(x)
x
O
y

6
(i)
2
( ) (4 ) ay x x = --- (1)
2 ( ) 4 2
dy
a ay x
dx
=
2 2
4 2 2
2
dy x x
dx a y a y

= =
For tangents // to the y-axis, is undefined
dy
dx
.
0 y =
Sub. 0 y = in (1), 0 (4 ) x x =
x = 0, x = 4

(ii) A = Area of PQRS = 2y (PQ)
2 (4 ) y x x = , 4 2 PQ x =
(symmetrical abt x = 2)

A =
2
8(2 ) 4 x x x

2
2
1 4 2
8 4 (2 )
2
4
dA x
x x x
dx
x x
(

= +
(




( )
2 2
2
8
4 (2 )
4
x x x
x x
(
= +



For 0
dA
dx
= ,
( )
2 2
4 (2 ) 0 x x x + =
2 2
4 4 4 0 x x x x + + + =

2
2 8 4 0 x x + =
2
4 2 0 x x + =
4 8
2 2
2
x

= =
Since x < 2, x = 2 2 .

Check maximum using first derivative test.
x
( 2 2 )

2 2 ( 2 2 )
+
f '( ) x +ve 0 ve



A is maximum when x = 2 2 .






7.
(i) ( )
( )( )
2
2
3 3 4
f
2 1 3
x x
x
x x
+
=
+









( )( )
( ) ( )( )
( )( )
( )
( ) ( )
( )
2
2
2
2 2
2
2
2
2
1
2
1
3 3 4
Let
2 1 3
2 1 3
3 3 4 3 2 1
1
Put 1
2
Equate coefficients of : 1
Equate constant terms : 1
3 3 4 1 1
Ans
2 1 3
2 1 3
1 2 1 3 1
3
1 1
x x A Bx C
x x
x x
x x A x Bx C x
x A
x B
C
x x x
x x
x x
x
x x

+ +
= +
+
+
+ = + + +
= =
=
=
+
= +
+
+
(
| |
( = + +
( |

\ .

= + ( )
( )( )
( )
( ) ( )
( )
2
2
2
2
2
1 2
2 2
2!
1
1 1 1
3 3
1 1
1 2 4
3 3 9
4 5 35
Ans
3 3 9
x x
x
x
x
x x x
x
x
(
+ +
(

(
| |
+ + +
( |
\ .

= + + +
= +

The expansion is valid when
( )
2
2 1 and 1 [1]
3
1
i.e. and 3
2
1 1
i.e. [2] Ans
2 2
x
x
x x
x
< <
< <
< <

( )
2 2
1
2
1 1
Coefficient of 2
3 3
1
2 Ans
3
n
n n
n
n
x
+
| |
=
|
\ .
| |
= +
|
\ .




9(b)
nth
birthday
Amount of Mrs Lees contribution ($) on her daughters
nth birthday
0 1000
1 1000(1.05)+1000
2 (1000(1.05)+1000)(1.05) + 1000 =
( ) ( )
2
1000 1.05 1000 1.05 1000 + +
8(i)


( )
5 4
f 1
1 1
x
x
x x

= = +


Since any horizontal line, y = k cuts the graph of f at at most 1
point,
f is 1-1. f
1
exists.
Let
4 4
1 1
1 1
y x
x y
= + =

.
( )
1
4 4
f 1 1 , \ {1}
1 1
x x
x x

= = + e


f
1
= f .

(ii)
( ) ( )
( ) ( ) ( ) ( )
51 50
2 1
f 4 = f f 4
f 4 f f f
5 4 1

1 4 3
x x x

(

= = =

= =


(iii)
( )
( )
2
2
g 2 4
= 2 1 2 2
x x x
x


= + +
+ + > +

For fg to be defined, R
g
_ D
f
= \ {1}
2 + > 1 > 3.
) ( | )
g f
7
2, 2 , , 1
3

|
+
|


.






9(a)
3 n n n
S S S =
3
2
n n
S S =
( ) ( )
3
2 3 1 2 1
2
n
a n n a n = ( (


7 1
2
n
a

=
3
( ) ( )
( )
( )
2
1000 1.05 1000 1.05 1000 1.05
1000
+ +
+

= ( ) ( ) ( )
3 2
1000 1.05 1.05 1.05 1
(
+ + +



17
( ) ( ) ( )
( ) ( )
17 16 15
2
1000 1.05 1.05 1.05
1.05 1.05 1

+ +

(
+ + + +


18
( ) ( ) ( )
( ) ( )
17 16 15
2
1.05 1000 1.05 1.05 1.05
1.05 1.05 1

+ +

(
+ + + +



Mrs. Lee: 1000(1.05), 1.05 a r = =
( )
18
1050 1.05 1
Total amount $
1.05 1
(


= $29500 (3 s.f.)

nth birthday Amount of Mr Lees contribution ($)
on his daughters nth birthday
1
( ) ( ) 1.05 500 1.05 500 + (


2
( ) ( ) ( )
2
1.05 2 1.05 500 1.05 500 +
= ( )
2
3 1.05 500
3
( ) ( ) ( )
2 3
1.05 3 1.05 500 1.05 500 +
= ( )
3
4 1.05 500

17
( ) ( )
17
18 1.05 500
18
( ) ( )
17
$1.05 18 1.05 500
(




( ) ( )
18
Total amount $ 18 1.05 500
(
=

= $21700 (3 s.f.)
10(i)
( )
( )
2
2 2
d 1
1 2
d
2 1 1
x
x x
x
x x
= =


( )
( )
1 1
2
1
2
1 2
1
1
1 1 2
-1
-1
1
cos d cos d
1
1 2
cos d
2
1
cos 1 Ans
cos d cos 1 Ans
x x x x x x
x
x
x x x
x
x x x C
x x x x x


(
=
(

= +
(
= =

} }
}
}

(a) ( )
1
cos 2 , 1 3 y x x

= s s



Translate the graph of
1
cos y x

= two units in the positive


direction of x-axis to obtain the graph of
( )
1
cos 2 y x

= .
Area of the region R =
( )
3
1
1
cos 2 d x x

= +
}

( )
1
1
-1
cos d where 2 ;
1 1 ; 3 1
2 Ans
t t t x dt dx
x t x t

= + = =
= = = =
=
}


(b) Volume generated
( )
( )
( )
( )
2
2
0 0
2
0
0
2
0
d cos 2 d
cos 4cos 4 d
1
cos 2 1 4cos 4 d
2
1 9 9
sin 2 4sin Ans
4 2 2

x y y y
y y y
y y y
y y y
= = +
= + +
(
= + + +
(

(
= + + =
(

} }
}
}



11(i) 1 d =
2
1 1
A x x A
y x
x x
+ +
= + =


1, 1 a b = =

(ii) When 0, x y c = =
When
2
0, 0
1
x x c
y
x
+ +
= =



1 1 4
2
1 1 4
2
c
x
c
+
=

+
=

Since 0 y < when 0, 0 0 x c c = < >

(iii)




y
x

O
0
3
R

Sketch graph of
( )
1
f
y
x
=
1 1
f( ) 1 x x
=


2
2 2
1
1
2 1
1
x x c
x
x
x x c x x
x c
+ +
=

+ + = +
= +

( )
1
1 1 4 1
2
c x + < < or
( )
1
1 1 4 1
2
c x c + + < < +

12















(i)
















16 16 12
0 , 8 , 8
4 6 7
OE OF OG
| | | | | |
| | |
= = =
| | |
| | |
\ . \ . \ .










(ii)
Area of AOEG =
1
2
OE OG =
16 12
1
0 8 16 21
2
4 7
| | | |
| |
=
| |
| |
\ . \ .

Area of AOEG =
1
( )( ) 16 21
2
OE d = , where d is the
perpendicular distance from G to OE
d = 8.89

(iii) Equation of the plane containing points E, F and G:
0 4
8 ; 8
2 3
0 4 4 1
4 8 4 4 1
1 3 16 4
1 16 1
equation of the plane is 1 0 . 1 32
4 4 4
EF EG
| | | |
| |
= =
| |
| |
\ . \ .
| | | | | | | |
| | | |
= = =
| | | |
| | | |
\ . \ . \ . \ .
| | | | | |
| | |
= =
| | |
| | |
\ . \ . \ .
n
r




(iv) From (iii), equation of plane containing the points E, F
and G is
1
1 32
4
| |
|
=
|
|
\ .
r .





i
j
k
o
o
o
O
E
F
G
16
8
7
A
B
C
D


Since
0
0
0
| |
|
|
|
\ .
does not satisfy the equation of plane EFG,
the points O, E, F and G do not lie on the same plane.

(v) The required plane H will be parallel to OF and the line
of intersection of planes OEG and EFG.
i.e. the plane H is parallel to OF and EG.
Thus, the equation is
8 4
4 8 ,
3 3
where , .


| | | |
| |
= +
| |
| |
\ . \ .
e
r

2009 HCI Preliminary Examinations H2 Mathematics Paper 2

No. Solution
1
( )
( ) ( ) ( )
( )
( )
2
d d
3
d d
d
3 3 3
d
d d
3
d 3 d
3
y u
x u
x x
u
x x u x x u
x
u x u x
x u u
x x x
x
=
(
=
(

= =



Method 1: (Partial Fractions)
( ) ( )
( )
( )
( )
( )
( ) ( )
2 2
2
d 3 1
d 3
3 3
3 1
d
3
3
3
ln 3
3
3 ln 3 3
u x
x x
x x
u x
x
x
x c
x
y x x c
= =

= + +

= + + (

}

Method 2: (Splitting of numerator)
( )
2 2
d 1 2 6 6
d 2 9 6
3
u x x
x x x
x
+ | |
= =
|
+
\ .

( )
2 2
d 1 2 6 6
d 2 9 6
3
u x
x x x
x
(

= + (
+

(


( ) ( )
2 1 6 3
ln 3 or ln 3
2 3 3
u x c x c
x x
(
= + + + +
(



( ) ( )
( ) ( )
2 1 3
3 ln 3
2 3
3 ln 3 3
y x x c
x
y x x c
(
= + +
(


= + + (


When x = 0, y = 0,
( ) 3 ln3 3 0 1 ln3 c c + + = =
( ) ( ) 3 ln 3 1 ln3 3 y x x = + (


As 3, 3 0 0 3 3 x x y + = .
Graph of ( ) ( ) 3 ln 3 1 ln3 3, 3 y x x x = + < (

.

2
1 3
Given 2 , 2
3 1
OA OB
| | | |
| |
= =
| |
| |

\ . \ .

y
x
Equation of l:
3
2 ,
1
m m
| |
|
= e
|
|
\ .
r .
Method 1:
Let F be the foot of perpendicular from C to the line l,
3
2 0
1
CF
| |
|
=
|
|
\ .
.
Thus,
3
( ) 2 0
1
OF OC
| |
|
=
|
|
\ .
3 3
2 2 . 2 0
1
m k
m k
m k
| | | |
| |
+ =
| |
| |

\ . \ .
3
9 3 4 4 0
7
m k m k m k m k + + + + = =
Since F is the mid-point of C and ' C , ' 2 OC OF OC =
=
11
1
2
7
13
k
| |
|
|
|

\ .
(Shown)
Method 2:
Let OF be the projection vector of OC on the line l.
Then
1 3 3
2 . 2 2
1 1 1
14 14
k
OB OB
OF OC
OB OB
| | | | | |
| | |

| | |
| | |
| |
\ . \ . \ .
= = |
|
\ .

=
3
3
2
7
1
k
| |
|

|
|
\ .

Since F is the mid-point of C and ' C , using Ratio Theorem
Since F is the mid-point of C and ' C ,
' 2 OC OF OC =
=
3 1 11
6 1
2 2 2
7 7
1 1 13
k k k
| | | | | |
| | |
=
| | |
| | |

\ . \ . \ .
(shown)
When BC is perpendicular to OA,
0
3 1
2 2 . 2 0
1 3
3 1
2 2 . 2 0
1 3
3 4 4 3 3 0
1
3
BC OA
k
k
k
k
k
k
k k k
k
=
( | | | | | |
( | | |
=
( | | |
| | |
(

\ . \ . \ .

+ | | | |
| |
=
| |
| |

\ . \ .
+ + =
=


When k =
1
3
,
1
3 11
1
2
and ' 2
3
21
13
1
3
OC OC
| |
| |
|
|
|
= =
|
|
|
|
\ .
|
\ .
.
Hence,

1
3 1 11
2
LHS = 6 15 6 2 15 2
3
3 13
1
3
11
RHS = 21 ' 2 LHS
13
OA OC
OC
| |
| | | |
|
| |
|
+ = + =
| |
|
| |
|
\ . \ .
|
\ .
| |
|
= =
|
|

\ .
(Verified)

Method 1:
6 15 21 '
6 6 ' 15 15 '
6( ') 15( ')
6 ' 15 '
OA OC OC
OA OC OC OC
OA OC OC OC
C A CC
+ =
+ =
+ =
=
0
0
0

Thus, ' C A is parallel to ' CC . Since ' C is the common point, the points A, C
and ' C are collinear.

Method 2:

6 15 21 '
6 15
'
21
OA OC OC
OA OC
OC
+ =
+
=

Hence by Ratio Theorem, we can deduce that A, C and ' C are collinear.
3(i) Complex No
(ii)

(iii)
(iv)
(v)

4
f(x) =
3
...
sin
3!
x
x
x
e e
+
=
2 3 4
3 3 3
3
3! 3! 3!
1 ...
3! 2! 3! 4!
x x x
x x x
x
x
| | | | | |

| | |
| |
\ . \ . \ .
= + + + + +
|
\ .

3 4 3 4
2
1 2
1 ...
6 2 6 6 24
x x x x
x x
| |
= + + + + +
|
\ .

=
2 4
1 ...
2 8
x x
x + + +

sin( ) sin cos cos sin sin sin( ) x x x x x
e e e e
+t t+ t
= = =
=
2 4
1 ...
2 8
x x
x + + [Replace x by x]

(i)
( )
2
2 2
2 2
2
3 2 2
3 2 2
d
4 1
d
Differentiating with respect to ,
d d d d
4 2( 1) 2 ( 1) (1)
d d d d
d d d d d d
2 ( 1) ( 1) (2)
d d d d d d
y
y
x
x
y y y y
y y
x x x x
y y y y y y
y y
x x x x x x
= +
= + = +
| |
= + + = + +
|
\ .

Sub x = 0, 1 y = ,
4 2 3 2
4 2 3 2
d d d d d d
2 ( 1) 2
d d d d d d
y y y y y y
y
x x x x x x
= + + +
( )
2
2 2
2 2
d d
4 1 1 1
d d
d d
2 (1 1)1 2 1
d
y y
x x
y y
dx x
= + =
= + = =

3 3
2
3 3
4 4
4 4
d d 3
2 (1 1)1 1
d d 2
d 3 d
2 1 (1 1) 2 3
d 2 d
y y
x x
y y
x x
= + + =
= + + + =

Using the Maclaurins formula,
g(x) =
2 3 4
3 3
1 ...
2! 2 3! 4!
x x x
x + + + + +
=
2 3 4
1 ...
2 4 8
x x x
x + + + + +
2 3 4 2 4
3 4
g( ) f ( ) 1 ... 1 ...
2 4 8 2 8
4
x x x x x
x x x x
x x
| | | |
= + + + + + + + +
| |
\ . \ .
+
~

(ii)
As x 0 ,
3 4
g( ) f ( ) 0
4
x x
x x
+
~ .
When x is close 0, the graph of g( ) y x = is close to the graph of f ( ) y x = OR
the graph of y=g( ) x is a good approximation to the graph of y=f ( ) x .
(iii)
Estimated error =
3 4 4 5 5
1
4 4 4 5 10
k
k
k
k
x x x x k
dx

( | | +
= + =
( |
\ .
}

5
2
10
10
k

<
5 1
10 k

< 0.631 k < 0 < 0.631 k < .
5(i) Let X denote the number of emergency telephone boxes in 1 km
2
. Then
Po(0.6) X .
(ii) Poisson distribution model is not suitable since the average rate of occurrence
of the emergency telephone boxes in the rural area (including the city) is not
constant throughout this area as city and rural area have different density for
distribution of emergency telephone boxes
6a) No.
Because not every resident will visit the shopping mall, or Consider total
population size, n, and take sample interval to be
50
n
instead of choosing every
10
th
shopper (or any other equivalent / logical reason).
(b) The surveyor is given quotas to fill. An example is the table shown below:
Gender
Male Female
25 25
The surveyor is given a free choice in picking the shoppers to fill the quotas.
(c) Increase sample size.
Interview people from different shopping malls.
Increase no. of categories or strata.
7 Case 1: Only 1 female does not know hip-hop
Prob =
10 8 12
2 1 1
20 20
2 2
C C C
C C

or
10 9 8 12
2
20 19 20 19

= 0.119668 or
216
1805

Case 2: Only 1 male does not know hip-hop
Prob =
8 10 10
2 1 1
20 20
2 2
C C C
C C

or
8 7 10 10
2
20 19 20 19

= 0.0775623 or
28
361

Case 3: All 4 people know hip-hop
Prob =
10 8
2 2
20 20
2 2
C C
C C

or
10 9 8 7
20 19 20 19

= 0.034903 or
63
1805

P(A)=
216
1805
+
28
361
+
63
1805
=
419
1805
or 0.232
P(B | A) = P(all 4 married | at least 3 know hip-hop)
=
P(all 4 married at least 3 know hip hop)
P(at least 3 know hip hop)


=
P(all 4 married all 4 know hip hop)
P(at least 3 know hip hop)


=
P(all 4 married )
P(at least 3 know hip hop)

=
2 4
2 2
20 20
2 2
419
1805
C C
C C
| |
|

\ .
| |
|
\ .
or
2 1 4 3
20 19 20 19
419
1805
| |

|
\ .
| |
|
\ .
= 0.000716

P(B) =
2 4
2 2
20 20
2 2

C C
C C
or
2 1 4 3
20 19 20 19
= 0.000166
Since P(B | A) = P(B), A and B are not independent events.
8


(i)
Assumption: The total no. of tickets in the box has to be large so that the
probability of getting a winning ticket is approximately constant after each
ticket is drawn.
Let X be the number of winning tickets in a draw of 5 tickets.
1
B(5, )
6
X
P( 1) X > = 1 P( 0) X = = 0.5981224

Method 1:
Required probability =
2
3 P( 1)[P( 0)] X X > =
=
2
3 (0.5981224)(0.4018776)
= 0.2897996 ~ 0.290

Method 2:

Let Y be the number of customers out of 3 with at least one winning ticket.
B(3, 0.5981224 ) Y
Required probability = P( 1) Y = = 0.2897996 ~ 0.29



ii) Since n = 60 is large, by the Central Limit Theorem,
5 5
~ N( , )
6 432
X approximately
Required probability = P( 1) X > = 0.0607
The event that all 60 customers each obtains at least one winning ticket is a
subset of the event that the average no. of winning tickets obtained by the 60
customers is at least one. Hence the probability is smaller.

9i) Case 1: I_ _ _ _ _ _ _ _ _ I:
No. of ways =
9!
2! 2!
= 90 720
Case 2: I_ _ _ _ _ _ _ _ _ O or O_ _ _ _ _ _ _ _ _ I:
No. of ways = 2
9!
2! 2! 2!
= 90 720
total no. of ways = 181 440
(ii)
No. of arrangements =
11!
3! 2! 2!

8!
2! 2!

= 1 663 200 10 080
= 1 653 120
(iii) The first letter T can be placed at the 1st, 2nd, 3rd, ..., 7th position.
The second T is then placed 4 positions after the first T.
No. of ways = 7
9!
3! 2!
1 = 211 679
(case where it is correct word is excluded)
10(i)

r = 0.972
Even though r is close to 1, it does not mean that there is a cause and effect
relationship between x and y.
(ii)


The points on the scatter diagram lie close to a straight line with positive
gradient. This agrees with the value of r obtained in part (i).
(iii)

For the model y = ab
x
, i.e. ln y = ln a + x ln b, r = 0.98759

For the model y = ax
b
, i.e. ln y = ln a + b ln x, r = 0.97789
Since | r | = 0.98759 is closest to 1 for the model y = ab
x
, this is the most
suitable model.
(iv) ln y = 0.128367x + 0.9994
ln a = 0.9994 a = 2.72
ln b = 0.128367 b = 1.14
11a) H
0
: = m
H
1
: > m
X ~ N(m, )
100
2
s
approximately.
Using z-test with n = 100,
x =
48005
100
+ 120 = 600.05; 600.05 100 60005 x = =


s
2
=
99
1
[36 990 985
2
60005
100
] = 9949.340909
10 5
x
10
5
y
Hence, Z =
9949.340909 / 100
X m
~ N(0,1)
To reject H
0
at 5% level of significance,

600.05
9949.340909 / 100
m
> 1.644853626
m s 584 (3s.f.)
b(i) If the claim that the mean SAT score has not improved is in fact true, 100p% of
tests using same method will lead to wrongly accepting the claim that the mean
SAT score has improved.
(ii) t-test should be carried out because sample size = 10 is small and population
variance is not given.
Assumptions:
The SAT score of students in the school follows a normal distribution.
The SAT scores of students are independent (or randomly selected) from one
another.
(iii) Not necessarily true that the same conclusion as t-test is reached (i.e., the null
hypothesis is not rejected) if z-test is carried out.










Reason:
For a fixed level of significance level o , z
o
< t
o
(refer to graph above).
When H
0
is not rejected for a t-test, there are two possible conclusions for a z-
test.
1) z
o
< test statistic < t
o
: H
0
will be rejected if z-test is used.
2) test statistic < z
o
< t
o
: H
0
will not be rejected if z-test is used.

OR









Let and
t z
p p be the p-values obtained using a t-test and z-test respectively.
Note that for the same test statistic, >
t z
p p
Since H
0
is not rejected for a t-test, p
t
>
100
o
. There are two possible
conclusions for a z-test.
1)
t
p >
100
o
>
z
p : H
0
will be rejected if z-test is used.
2)
t
p >
z
p >
100
o
: H
0
will not be rejected if z-test is used.






12(i) Let T be the travel time taken by Andy to reach the pick-up point.
2
N(18, 4 ) T
P(Andy takes bus B) = P(15 25) T < s
= 0.733313 ~ 0.733

(ii) P(Andy misses the buses) = P( 25) T > = 0.040059
Let X be the number of times in 8 work weeks that Andy misses the bus.
B(40, 0.040059) X
test statistic
N(0,1)
t( )
N(0,1)
t( )
Since n is large and 1.60236 5 np = < ,
Po(1.60236) X
Required probability = P( 4) X >
= 0.0791
(iii)
Let
2
T be the average time spent on travelling to the pick-up point on the first
two days of the week and
3
T be the average time spent on travelling to the
pick-up point on the last three days of the week.
2
2
4
N(18, )
2
T and
2
3
4
N(18, )
3
T
2 3
40
N(0, )
3
T T
2 3
P(0 5) 0.414548 T T < < =
~ 0.415
(iv) Let the number of minutes that Andy has to leave the home earlier be a
minutes.
P( 15 ) 0.90 T a s + > or P( ) 0.90 T k s >
From GC, P( 23.126) 0.90 T s =
Hence 15 23.126 a + >
8.126 a >
Thus he should leave his home at least 9 mins earlier.
i.e his latest time to leave home is 6.51 a.m.


2009 MJC H2 MATHS (9740) JC 2 PRELIM EXAM P1 SOLUTIONS

Qn

Solution
1 MOD

2
2 1 1
6 8 2 4 r r r r
=
+ + + +


( )
2
1 1
1 1
5 3
1 1
4 6
1 1
1 3
1 1
2 4
1 1 1 1
3 4 3 4
7 1 1
12 3 4
2 1 1
6 8 2 4
n n
r r
n n
n n
n n
n n
r r r r
= =
+ +
+ +
+ +
+ +
| |
=
|
+ + + +
\ .
=
+
+
+
= +
= +



( )
( )
2
2 2
1 1
2
1
7 1 1
12 3 4
70 1 1
3 3 4
6 88 80
1
6 8 6 8
2
40
6 8
40
40
n n
r r
n
r
n n
n n
r r
r r r r
n
r r
n
n
= =
=
+ +
+ +
+ + | |
= +
|
+ + + +
\ .
= +
+ +
(
= + +

= + +


2 Mathematical Induction Recurrence Relation

Let
n
P be the statement
2
1
1
1

=
+ n
n
x
S for all
+
Z e n .
To prove
1
P is true:
2
1
1
1
1
1
: LHS
1
1
1
1
=

=
x
x
S
i i

LHS
2
1
2
1
1
2 4 3
1
1
2
1
1 : RHS
1
2
1
2
= = =
+
=

x x
x
(sub in 3
1
= x )

true. is
1
P

Assume i.e. , some for true is
+
Z e k P
k
2
1
1
1

=
+ k
k
x
S .
To prove i.e. , true also is
1 + k
P
2
1
1
2
1

=
+
+
k
k
x
S .








( )( )
RHS.
2
1
1
2 4 3
1
1
2 3
1
1
1 2
2 1
1
1
1
2
1
1
1
1
2
1
1
1
1
1
1

1
1
: LHS
2
1
2
1
1
2
1
1 1
1 1
1 1
1 1
1 1
1
1
1
=
|
|
.
|

\
|

=
|
|
.
|

\
|
+
=
|
|
.
|

\
|
+
=
(


+
=
|
|
.
|

\
|

=
+
+ +
+ +
+ +
+ +
+ +
+ +
= +
+
=
+

k
k k
k k
k k
k k
k k
k k
k
i k i
k
i i
k
x
x x
x x
x x
x x
x x
x x
x x
x
S



1
1
1
is true is also true.
Since is true
and is true is also true,
by Mathematical Induction, is true for all .
k k
k k
n
P P
P
P P
P n
+
+
+

eZ


Q3 Integration by substitution


}

x
x x
x
d
2 2 ln ln
2 ln ln

t
x e 2 =

}

= t
t
x
t t
t
d
d
d
2 2 ln e 2 ln e 2
2 ln e 2 ln

t
t
x
e 2
d
d
=
}
+
+
= t
t
t
t
t
d e 2
2 2 ln 2 ln e 2
2 ln 2 ln

}

= t
t
t
d
2

( ) 2 2
d
2
t
t
t
+
=

}

( )
1
2
2
2 d
2
t t
t
= +

}
(shown)


}

4
2
e 2
e 2
d
2 2 ln ln
2 ln ln
x
x x
x

( )
}

+ =
4
2
2
1
d
2
2
2 t
t
t
( ) ( )
4
2
2
1
2
3
2 4 2
3
2
(

+ = t t
( ) ( )
(

+ = 2
1
2
3
2 4 4 2 4
3
2

( ) ( )2
1
2
3
2 4 2
3
2
+ =
( ) ( )
(

+ = 4 2
3
2
2 2
1

3
2 16
=

Therefore 16 = a , 3 = b
Q4 DE

Let B denote the number of births and D denote the number of deaths.
d d
Given
d d
B B
x ax
t t
=
and
2 2
d d
d d
D D
x bx
t t
= where a, b are positive constants
2
2
2
d d d
d d d
d
(shown)
d
d
Given that 0 when 200,
d
200 200 0
200
d
(200 )
d
x B D
t t t
x
ax bx
t
x
x
t
a b
a b
x
ax bx bx x
t
=
=
= =
=
=
= =


1
d d
(200 )
x b t
x x
=

} }

( )
1 1 1
d
200 (200 )
1
ln ln(200 )
200
1
ln
200 200
x bt d
x x
x x bt d
x
bt d
x
(
+ = +
(


= + (

| |
= +
|

\ .
}


ln
200
e
200
e , e
200
200
e
200
e 1
kt r
kt r
kt
kt
x
kt r
x
x
x
x
A A
x
x
B
x
x
B
+

= +

= =

=
=
+



As
200
, 200
e 1
kt
t x
B

=
+


Hence there is a limit to the size of the population. (shown)

200 is the maximum number of ToTo bears the environment can support.
Q5 Vectors Exclude Planes and Lines
(i)







(ii)













(iii)


Since AB = b a and A, B and C are collinear,
2 k =

2 2
3 2
OC OB BC = +
=

b+ b a
= b a


2
2
3
4
3
4
4
16
3
4 4 3
3 3
. =
=
=
= =
a
a b
a
a
a
a


Alternatively,














(iv)
( ) ( )
^ ^
2
2
2
2
3
4
4
3
4
3
4
4
16
3
4 4 3
3 3

| |
= =
|
\ .
=
=
=
= =
b a a
a b a a
a
a
a
a
a
a
a



( )
2 2 2
0
b 2 2 2 2 3
a.b
cos
a b
4
4 3
2 3
3
1
2
or 60
3
u
t
u
= + + =
=
=
| |
|
\ .
=
=





















A
B
C O
N
D
E


( ) ( )
2
3
2 2 3
3
2
2
2
DC DA
DE
+
=
+
=
=
| |
|
|
|

\ .
a a b
b
=


Alternatively,

( )
2 2
3 2 2 2
3 3
2 3 2 2
2
2
2
AE AE
DE DA AE CO AE
= = =
= + = +
= +
=
| |
|
=
|
|

\ .
b a a b a
a b b a
b

Q6 Complex Numbers Loci
(i)
(ii)
(iii)
(iv)
Q7 Complex Numbers Exclude Loci
(a)
(b)
Q8 Binomial Expansion and Maclaurins Series
(a) Method 1
Let
( )
f ( ) 1 e
n
x
x = + .
( ) ( )
1
f ( ) 1 e e
n
x x
x n

' = +
( )( ) ( ) ( ) ( )
2 2 1
f ( ) 1 1 e e 1 e e
n n
x x x x
x n n n

'' = + + +
Therefore, f (0) 2
n
= ; ( )
1
f (0) 2
n
n

' = and ( )( ) ( ) ( )( )
2 1 2
f (0) 1 2 2 1 2
n n n
n n n n n

'' = + = +


( )
( )
2
1 2
1 3 2
1 2
f ( ) 2 2 ...
2!
2 2 1 2
n
n n
n n n
n n
x n x x
n x n n x


( +

= + + +
( ~ + + +



Method 2
Using standard series
2
e 1
2
x
x
x = + + (taking the first three terms only)
2 2
1 1 2
2 2
n n
x x
x x
| | | |
+ + + = + +
| |
\ . \ .


2
2 1
2 4
n
n
x x | |
= + +
|
\ .


( )
2
2 2
1
2 1
2 4 2 2 4
n
n n
x x x x
n
(
| | | |
= + + + + + (
| |
( \ . \ .



( )
2
2
1
2 1
2 4 2 2
n
n n
n n x
x x
(

| |
= + + + +
(
|
\ .
(



( )
2
1
2 1
2 4 8
n
n n
n n
x x
( | |
= + + + +
( |
(
\ .


( )
2
3
1
2 1
2 2
n
n n
n
x x
( + | |
= + + +
( |
(
\ .

( )
1 3 2
2 2 1 2
n n n
n x n n x

( ~ + + +






(bi) Method 1
3 2
x
y
x
=


( )
( )
( )( )
1
1
2
2
2 3
(3 2 )
2
(1 )
3 3
1 2
2 2
1 1 ...
3 3 2! 3
2 4
1 ...
3 3 9
1 2 4
...
3 9 27
y x x
x
x
x
x x
x
x x
x x x

=
=
| |

| | | |
= + + +
|
| |
|
\ . \ .
\ .
| |
= + + +
|
\ .
= + + +


Method 2
( ) 3 2
1 3 1
2 2 3 2
x
y
x
x
=

| |
= +
|

\ .

( )
1
1 3 1
2 2 3 2
1 3
3 2
2 2
x
x

| |
= +
|

\ .
= +


( )
( )( ) ( )( )( )
1
2 3
2 3
2 3
1 3 1 2
1
2 2 3 3
1 2 1 2 3
1 1 2 2 2
1 1 ...
2 2 3 2 3 3 3
1 1 1 2 4
...
2 2 3 9 27
1 2 4
...
3 9 27
x
x x x
x x x
x x x

| || |
= +
| |
\ .\ .
| |

| | | | | |
= + + + +
|
| | |
|
\ . \ . \ .
\ .
= + + + + +
= + + +

(bii)
2
1
3
x <
Therefore the set of values of x =
3 3
: ,
2 2
x x x

e < <
`
)

(biii
)
3 2
x
y
x
=


2 3
1 2 4
...
3 9 27
x x x = + + +

Clearly the coefficient of
n
x follows a GP with first term
1
3
and common ratio
2
3

Therefore coefficient of
n
x =
1 n
n
T ar

= =
1
1 2
3 3
n
| |
|
\ .

Let f ( ) y x = .
( ) ( ) ( )
( )
( )
( )
n
2
f 0 f 0
f f 0 f 0
2! !
n
x x x x
n
''
' = + + + + +
By considering the coefficient of
n
x for the Maclaurins expansion, at 0 x = ,

( )
( )
1 n
f 0
1 2
! 3 3
n
n

| |
=
|
\ .

( )
( )
1
n
1 2
f 0 !
3 3
n
n

(
| |
=
(
|
\ .
(

.

Q9 Vectors Planes and Lines

(i)






(ii)












(iii)










Sub ( ) , , 0 o | into H
1
and H
2
.
1
2
: 1( ) 3( ) (0) 8
: 3( ) 1( ) (0) 0
a
b
t o |
t o |
+ + =
+ + =


Using GC, 1, 3 o | = =

1 3 3
3 1 3
8
b a
a b
a b
| | | | | |
| | |
=
| | |
| | |

\ . \ . \ .


1
1 3
: r 3 3 ,
0 8
b a
l a b
| | | |
| |
= + e
| |
| |

\ . \ .




1 3
3 3
0 8
b a
a b
| | | |
| |
+ =
| |
| |

\ . \ .
5
2
2
| |
|
|
|
\ .
+
4
1
0

| |
|
|
|
\ .


1
4
=

Sub a b = and
1
4
= , we have












(iv)

( )
( )
1
1 4 5 4
4
1
3 4 2
4
a
a

= +
= +



2
2
a
b
=
=



Since the 3 planes have no point in common, l
1
cannot intersect H
3
.

1
1 1
: r 3 1 ,
0 1
l
| | | |
| |
= + e
| |
| |

\ . \ .


Condition 1:
( ) 1, 3, 0 is not on H
3
.
Hence,
( 1) (3) (0) 11
4
p q
p
+ + =
=


Condition 2:
1
p
q
| |
|
|
|
\ .
and
1
1
1
| |
|
|
|

\ .
are perpendicular.

1 1
1 0
1
1 0
1
. p
q
p q
p q
| | | |
| |
=
| |
| |

\ . \ .
+ =
=



Q10 AP, GP
(a)
Given
( )
( )
3
3
1
26 26
1 0, 1
1 27 1 27
a r
a
r a r
r r

| |
= = = =
|

\ .


3
1
27
1
3
r
r
=
=

Since we have ( )
2
1 ar ar d ar r d = =

1 2
3 3
2

9
a d
a d
| |
=
|
\ .
=


Therefore the sum of the first fifty-five terms of the A.P.

55 2
2 54 275 (Ans)
2 9
a a a
( | |
= + =
| (
\ .


(b) Arithmetic Progression with first term,
1
30 T = , common difference, 5 d = , in minutes.

(i) ( )
7 1
6 30 6 5 60 T T d = + = + = minutes = 1 hour.
(ii)
| |
7
7
30 60 315
2
S = + = minutes = 5.25 hours
(iii) To determine the n
th
lesson, where Ann would have completed 60 hours (3600
minutes) of lesson. Note n must be a positive integer.
( ) ( ) 2 30 ( 1) 5 3600
2
n
n + > (


| | 55 5 7200 n n + >

2
11 1440 0 n n + >
32.8 or 43.8 n n > s
(Note that students can use GC to get n)

Hence during the 33
rd
lesson, Ann will have completed a total of 60 hours.


Alternatively,
( ) ( ) 2 30 ( 1) 5 3600
2
n
n
S n = + > (


Using GC,
when n = 32, 3440 3600
n
S = <
when n = 33, 3630 3600
n
S = >
Hence during the 33
rd
lesson, Ann will have completed a total of 60 hours.


Q11 Differentiation (Applications and Graphs of f)
a














b d d
sin ; cos .
d d
d
cot
d
x y
a t a t
t t
y
t
x
= =
=


x
0 B (-3, 0)
A (-1,0)
x = -2
y
y = 1




When
2
a
x = ,
3
t
t
= and
3
2
a
y =


Equation of tangent:

( )
3 3
2 3 2
3 3 3
3 3 2 2
3 3 4
3 2 3
3 3 2
a
y a x
a
y x a
x a
y x a
| |
=
|
\ .
= + +
| |
= +
|
\ .
= +

At x = 0:
2 3
3
y a =
At y = 0: 2 x a =
Therefore, area of triangle OQR = ( )
2
1 2 3 2 3
2
2 3 3
a a a
| |
=
|
|
\ .

C Let the internal height be h, external length be Y and the external height be Z.
Volume = 1000
3
cm
2
2
1000
1000 x h h
x
= =

Also, 2 Y x = + and 1 Z h = +
Material needed,
2 2
S Y Z x h =
( )
( )
2
2
2 2
2
2
1000 1000
2 1
1000
4 1 1
x x
x x
x x
x
| | | |
= + +
| |
\ . \ .
| |
= + + +
|
\ .

( )
3 2
4 3
d 2000 1000
2 4 1 4 1 0
d
2 4 4000 8000 0
2 (rejected) or 12.6 cm
S
x x
r x x
x x x
x x
| | | |
= + + + + =
| |
\ . \ .
+ =
= =


x
12.599


12.599
12.599
+

d
d
V
x


Answers for non-contextual questions have to be exact when using the test.
2009 MJC JC2 Preliminary Exam
H2 Mathematics (9740) Paper 2 Suggested Solution

Qn Solution
1 Integration by parts
(i)
.
(ii)






Qn Solution
2 Graphs
(a)
















(b)(i)




















( ) f 2 1 y x =







cos cos
d
e e sin
d
x x
x
x
=
cos
e sin 2 d
x
x x
}
( )
cos
e 2sin cos d
x
x x x =
}
( )( )
cos
e sin 2cos d
x
x x x =
}
cos cos
2e cos 2 e sin d
x x
x x x =
}
cos cos
2e cos 2e
x x
x C = + +
x
y
4 y =
1
,
2
a
| |
|
\ .
3
2
x =
( ) f 2 1 y x =
1
, 0
2
b + | |
|
\ .
1
, 0
2
b + | |
|
\ .
5
2
x =
y
x
O
3 x =
0 y =
( ) 8, 0.2
11
9
11
2

( )
2
2 11
3
x
y
x
+
=
+
x
y
4 y =
( ) 1, a
3 x = 5 x =
( ) f 1 y x =
( ) 1, 0 b + ( ) 1, 0 b +



(ii)
























Qn Solution
3 Equations (System of equations) and Inequalities
(a)
2
2
d
2
d 2
b
y a x cx d
x
y a b
cx
x x x
= + +
= +

Augmented matrix
1 1 1 1 9
0.5 1 2 0 0
1 23
2 16 1
4 4
1 217
3 81 1
9 3
| |
|

|
|

|
|
|

|
\ .
OR
9
1
2 0
2
1 23
2 16
4 4
1 217
3 81
9 3
a b c d
a b c
a b c d
a b c d
+ + =
+ =
+ + =
+ + =

Using GC, 2, 3 1 15 a b c d = = = =
The equation of the curve is
2
3
2 15 y x x
x
= + +
(b)
(i)
( )
2
2
4 4 3 2 1 2 0 x x x x + + = + + > e

Alternative Method
Discriminant = ( )( )
2
4 4 4 3 32 =
Since coefficient of
2
x is positive and discriminant < 0,
( )
2
f y x =
( )
0, a
( )
0, a
( ) , 0 b ( ) , 0 b
4 x = 4 x =
2 y =
2 y =
y
x
2
4 4 3 x x + + is positive
(ii)





Therefore or








(iii)
( )
( )
3 2
2
4 ln 4 ln 3 ln
0
ln ln 2
x x x
x x
+ +
>
+


Replace by
The solution is
or

Qn Solution
4 Functions
(i)











Either:
Any line cuts the graph twice; hence f is not 1-1 and does not exist.
OR:
3 y = cuts the graph twice; hence f is not 1-1 and does not exist.

The largest value of k is 1.
( )
( )( )
( )( )
( )
3 2
2
2
2
4 4 3
0
2
4 4 3
0
2 1
0 since 2 1 2 0
2 1

x x x
x x
x x x
x x
x
x x
x x
+ +
<
+
+ +
<
+
< + + > e
+
2 x < 0 1 x < <
x ln x
2 ln 0
ln 0 ln 0
1
x
x x x
x
+
< s
= > e
=
1
ln 1
ln 1 or ln 1
0 e or e
x
x x
x x

>
< >
< < >
, 2 y k k = >
1
f

1
f

0 1
+


+
-2


O



x
y
( )
| ) 1
2
2
2
2
2
1
f
f
2
1
1
Let ,
1 0
4
2
4
Reject 0 1.
2
4
2
4
f
2
D R 2,
4
f : , 2.
2
y x
x
x xy
y y
x
y y
x x
y y
x
x x
x
x x
x x

= +
+ =

=
+
= < s

=

=
= =

>



For hg to exists, .
, .
Hence, the minimum value of is 9.

To find the ,

Method 1



Method 2











From the graph,
.


Qn Solution
5 Integration Area and Volume
(a)
( )
2
2 2
x y a a + =
( )
2
2 2
x a y a =
g h
R D _
( |
g
R 9, = ( )
h
D 0, =

hg
R
( ) ( | ( |
g g h hg
D 2, 2 R 0, 9 D R ln2, ln11 = = = =
( |
hg
R ln2, ln11 =


Volume formed ( )
2
2
2
0
d
a
a y a y t
(
=

}


( )
2
2 2 2
0
2 d
a
a y ay a y t
(
= +

}
OR ( )
2
3
2
0
1
3
a
a y y a t
(
=
(



( )
2
2 2 2
0
2 d
a
a y ay a y t
(
= +

}

( )
3 3 3
1 1
2 0
3 3
a a a t
( | | | |
=
| | (
\ . \ .


2
2
0
2 d
a
y ay y t ( = +
}

3
4
3
a t =

2
3
2
0
3
a
y
ay t
(
= +
(



3
3
8
4
3
a
a t
(
= +
(



3
4
3
a t =
Volume of sphere with radius a is
3
4
3
a t =
Therefore volume of the semi-circle obtained when rotated 2t radian about the y-axis is
equal to the volume of a sphere with radius a


When
|
|
.
|

\
|
=
2
3
ln x 2 = t

|
|
.
|

\
|
=
5
24
ln x 5 = t
( )
( ) t t
t
t
x ln 1 ln
2
1 1
ln
2
2
1
2
=

=


t t
t
t
x 1
1
2
2
1
d
d
2

|
.
|

\
|

=

( ) 1
1
d
d
2

=
t t t
x

Area of required region
}
=
5
24
ln
2
3
ln
dx y

}
=
5
2
d
d
d
t
t
x
y
( )
( )
}

=
5
2
2
2
d
1
1
8 5 t
t t
t t

( )
}

=
5
2
2
2
d
1
8 5
t
t
t


( )
}

=
5
2
2
d
1
3
5 t
t


5
2
3 1
5 ln
2 1
t
t
t
( | |
=
| (
+
\ .


3 4 3 1
25 ln 10 ln
2 6 2 3
( (
=
( (



3 1
15 ln
2 2
= + OR
3
15 ln 2
2
=


Qn Solution
6 Sampling Methods
A sample is a small representative of the population.
In many practical statistical investigations, it may not be possible to collect information for
the entire population due to time or cost constraints (or any other sensible reasons). Hence
it is desirable to use the results obtained from a sample to make statistical inferences about
the population.
Any sensible real-life example which gives a complete sampling frame and ensures that
every member of the population has an equal chance of being selected.
Qn Solution
7 Hypothesis Testing

Unbiased estimate of population variance = ( )
2 80
11.23
79

= 127.71

H
0
: 1000 =
H
1
: 1000 =

Since n = 80 is large, by Central Limit Theorm,

2
~ ,
s
X N
n

| |
|
\ .
approximately

Test statistic :
X
Z
S
n

=

Level of significance: 4%

Critical Region: Reject H
0
if p-value < 0.04

Assuming H
0
is true, from the graphic calculator, p-value = 0.0687

Conclusion: Since p-value = 0.0687 > 0.04,
we do not reject H
0
and conclude that there is no significant evidence,
at 4% level, the mean volume is not 1000 ml.




















Qn Solution
8 Permutation and Combination
(a)(i)


1 B, 2 E, 1 A, 3 N, 3 T

8!
No. of ways 1680
3!2!2!
= =
(ii)

Case 1: _B_E_E_A_
Slot in TTT first in
5
4!
3 2!
| |

|
\ .

_T_B_E_T_E_T_A_
Follow by slotting in NNN.
So no. of ways for case 1 =
5 8
4!
3 3 2!
| | | |

| |
\ . \ .
= 6720

Case 2: _B_E_E_A_
Slot in TTT first in
5
4!
2!
2 2!
| |

|
\ .
2! Because TT and T can swop.
_T_B_E_T_T_E_A_
Follow by slotting in NNN (1 N must slot in between TT)
So no. of ways for case 2 =
5 7
4!
2!
2 2 2!
| | | |

| |
\ . \ .
= 5040

Case 3: _B_E_E_A_
Slot in TTT first in
5
4!
1 2!
| |

|
\ .

_B_E_T_T_T_E_A_
Follow by slotting in NNN (2 N must slot in between TTT to separate them)
So no. of ways for case 3 =
5 6
4!
1 1 2!
| | | |

| |
\ . \ .
=360.
Total number of ways = 6720 + 5040 +360 = 12 120

(b)
(i)



No. of ways =
5 10
No. of ways 8! 42336000
4 4
| | | |
= =
| |
\ . \ .



(ii)
3 4 10 9
8!
No. of ways 65318400
1 2 1 2 2!2!
| || || || |
= =
| | | |
\ .\ .\ .\ .




Qn Solution
9 Probability
(a)
(i)

P(all three cards even)
10 9 8 2
20 19 18 19
= = or
10
3
20
3
2
19
| |
|
|
\ .
| |
|
|
\ .
= =
(ii)

P(exactly one even)
10 10 9 15
20 19 18 38
3
1
| |
= =
|
\ .
or
10 10
1 2
20
3
15
38
| || |
| |
| |
\ .\ .
| |
|
|
\ .
= =
(b)
































(i)

(ii)


(iii)
( )
16 16 15 4 4
5 20 19 20 19
P B = + =
15 16 1 4
20 19 20 19
79
380
P( ) P(1st card 5 & 2nd card 5)
P(1st card 5 & 2nd card 5) P(1st card 5 & 2nd card 5)
A B = s >
= = > + < >
= +
=
( )
( )
79
P( )
380 79
95 1
P
4
P
A B
A
B A

= = =


Using tree diagram:








( )
16 15 15 15 4 1 4
5 20 19 20 19 20 19
P B = + + =
16 15 79 4 1
20 19 20 19 380
P( ) A B = + =
( )
( )
79
P( )
380 79
95 1
P
4
P
A B
A
B A

= = =









B ( 5)
B ( 5)
B ( 5)
B

(< 5)
B

(< 5)
B

(< 5)
1 4
5
6 20























Qn Solution
10 Binomial and Poisson Distributions (include approximation)
(i) Let C and P be the number of cheese and peanut waffles sold in a randomly chosen 15-
minute period respectively.
C ~ P
o
(4) , P ~ P
o
(5)
C + P ~ P
o
(4 + 5) P
o
(9)

P( 10) 1 P( 9) C P C P + > = + s
= 0.41259
0.413 ~
(ii)
Required probability = ( ) P all waffles sold are peanut flavour | at least 10 waffles are sold


( )
( )
P all waffles sold are peanut flavour and at least 10 waffles are sold
P at least 10 waffles are sold
=

P( 10) P( =0)
P( 10)
P C
C P
>
=
+ >

=
| | 1 P( 9) P( =0)
P( 10)
P C
C P
s
+ >

=
( ) ( ) 0.031828 0.018316
0.41259


= 0.00141
(iii)



P(at least 10 waffles are sold in each of the 4 consecutive 15-minute period in a randomly
selected hour) = ( )
4
0.41259 or ( )
4
0.413
= 0.0290 or 0.0291 (to 3 s.f.)

Alternatively
Let T be the number of 15-minute periods in a randomly chosen one-hour period where at
least 10 waffles are sold.
T ~ B(4, 0.41259)
Required probability P( 4) T = = = 0.0290 or 0.0291 (to 3 s.f.)
(iv) Let X be the number of waffles sold in an hour.
X ~ P
o
(9 4) ( )
o
P 36 X
Since 36 10 = > , ( ) N 36, 36 X approximately
( ) ( ) P 40 P 39.5 X X > = > after continuity correction
= 0.280 (to 3 s.f.)

Let A be the event that at least 10 waffles are sold in each of the 15 minute period in an
hour.
Let B be the event that at least 40 waffles are sold in an hour.
Obviously, event A is a subset of B and hence the probability of A is smaller than that of B
Qn Solution
11 Correlation and Regression
(a)(i) 0.89241 0.892 r = = (3 s.f.)
(a)(ii) 0.95956 0.960 r = = (3 s.f.)
Since 0.960 r = is closer to 1 than 0.892 r = , the model in (i) is less suitable than the
model in (ii).
(a)
(iii)
Regression line of F on x: 0.35903 0.029245 F x = +
0.359 0.0292 F x = +
Regression line of x on F: 204.51 31.484 x F = +
205 31.5 x F = +
(a)
(iv)
Using 0.35903 0.029245 F x = + ,

2
100 0.35903 0.029245t = +
58.37047 58.4 t = = s (3 s.f.)
(b)
(i)
Regression line of x on y : 3 2 51 x y + =
51 2
3 3
x y =
Regression line of y on x : 8 11 36 x y =
36 8
11 11
y x = +
Since
8 2
11 3
bd
| || |
=
| |
\ .\ .
is negative and
2
r is always positive, therefore
2
bd r = .
Hence 8 11 36 x y = is not valid.
OR
Since b and d must be both positive or both negative, but
8
11
b
| |
=
|
\ .
and
2
3
d
| |
=
|
\ .
,
hence 8 11 36 x y = is not valid.






(b)
(ii)







From
51 2
3 3
x y = , we have
2
3
d = .
Gradient of regression line of x on y in graph of y against x is
1 1 3
2
2
3
d
= =



Since 0.943 r = ,
Using
2
bd r = ,
x
y




2
2
( 0.943)
3
3
1.33
2
b
b
| |
=
|
\ .
= >

Therefore regression line on y x has more gentle slope than regression line on x y .











Qn Solution
12 Normal Distribution (include Sample Mean CLT)
(a)
By symmetry,
155 185
170
2

+
= =
( ) P 155 0.025
155 170
P 0.025
15
1.95996
7.6532
X
Z
o
o
o
< =
| |
< =
|
\ .
=
=

2
58.6 o = (3 s.f.)
(i)
( )
2
1 2 3
2 ~ 0, 6 X X X N o +
( ) ( ) ( ) ( )
1 2 3
E 2 E E 2E 0 X X X X X X + = + =
( ) ( ) ( )
2
1 2 3
Var 2 2Var 4Var 6 X X X X X o + = + =

( )
1 2 3
P 2 5 X X X + > +
( )
1 2 3
P 2 5
0.39484
X X X = + >
=

= 0.394 (3 s.f.)

(ii)
2
~ ,
50
X N
o

| |
|
\ .
(Since X is normally distributed, there isnt a need to use CLT.)
( )
P 172
0.96769
X <
=

= 0.968 (3 s.f.)


(b) Since n is large, by CLT
2
5
~ , X N
n

| |
|
\ .
approx.
( )
P 1 0.99
1
P 0.99
5
1 1
P 0.99
5 5
1
1 2P 0.99
5
1
P 0.005
5
0.2 2.5758
0.2 2.5758
165.87
X
Z
n
Z
n n
Z
n
Z
n
n
n
n
< >
| |
|
< >
|
|
\ .
| |
|
< < >
|
|
\ .
| |
|
< >
|
|
\ .
| |
|
< <
|
|
\ .
<
>
>

Least n = 166.



2009 RJC H2 Maths Preliminary Examination Paper 1 (Solutions)

1


From the graphic calculator, the x-coordinate of the 3 points of intersection are:
4.4140, 1.3005 and 6.4352. (5 s.f.)
Now,
3 2
1 1
1 2 5
12 6
x x x x + s +

3 2
1 1
3 1 6 15
4 2
x x x x + s + .
Hence, from the graph, 4.41 1.30 or 6.44 s s > x x . (3 s.f.)

2

(i)













(ii)

2
e 3 5 (1)
x
y y + + =

Differentiating (1) with respect to x,
d d
2 e 3 0
d d
d
(2 3) e 0 (2)
d
x
x
y y
y
x x
y
y
x
+ + =
+ + =


Differentiating (2) with respect to x,
( )
2
2
2
2
2
d d d
2 (2 3) e 0
d d d
d d
2 2 3 e 0 (Shown) (3)
d d
x
x
y y y
y
x x x
y y
y
x x
| || |
+ + + =
| |
\ .\ .
| |
+ + + =
|
\ .



2
e 3 5 (1)
x
y y + + =

When x = 0,
2
3 4 0
( 1)( 4) 0
y y
y y
+ =
+ =
(from (1))
y = 1 or 4 (NA since y > 0)


d 1
When 0,
d 5
y
x
x
= = (from (2))

2
2
d 27
d 125
y
x
= (from (3))

So
2
1 27
1 ...
5 250
y x x = +















3


15
y


x



0






( 4.41, 10.2)



(1.30, 6.90)

(6.44, 22.3)

3 Floor area =
( )( )
1
4 2 2 sin
2
u
| |
+
|
\ .

4 2sinu = +

Unit cost of carpeting =
8000
4 2sinu +

[MF15(under Maclaurins series) : sin ... u u = +
Since u is small, sinu u ~ (as the third and higher powers of u are negligible).]

( )
( )
1
1
2
8000
4 2
4000 2
1
4000 1
2 2
2000 1 (Binomial series)
2
$ 2000 1000 /m
u
u
u
u
u

~
+
= +
| |
| |
= +
|
|
|
\ .
\ .
| |
~
|
\ .
=

So a = 2000, b = 1000

4(i)





(ii)

3 2
3
sec tan d (sec tan )(sec ) d
1
sec
3
x x x x x x x
x c
=
= +
} }


Let u x = . Then
d
2
d
x
u
u
=

2
2
1
d
2 ( 9)
1
(2 ) d (I)
2 ( 9)
1
d
9
1 3
ln
6 3
1 3
ln +c
6 3
x
x x
u u
u u
u
u
u
c
u
x
x

= +
+

=
+
}
}
}

5(i)





Since 2(2) 0 2 2 = , the point with position vector 2 2 + i k lies on H
2
.

A direction vector for the line of intersection is
1 2 5
3 1 5
2 1 5
| | | | | |
| | |
=
| | |
| | |

\ . \ . \ .
.


An equation of line of intersection, l , is
2 1
0 1 ,
2 1

| | | |
| |
= + e
| |
| |
\ . \ .
r

5(ii) From GC
3 2 6
2 2
3 12
6, 4, 6
x y z
x y z
x y z
x y z
+ =

= = =

The 3 planes intersect at a common point with position vector
6
4
6
| |
|
|
|
\ .
.

5(iii)
For the 3 planes to intersect at a common line, line l must lie on plane H
3
.
2 1 1
0 1 4
2 1 1
(2 ) (2 ) 4
4
4 or 0
b b
b b
b b
b

( | | | | | |
( | | |
+ =
( | | |
| | |
(

\ . \ . \ .
+ + + =
=
= =
--- (*)
Equation (*) must be true for all real values of .
So b = 0.

Alternative Method 1
2 1
0 4 0
2 1
b b b
| | | |
| |
= =
| |

\ . \ .
and
3 1
1 4 4 0
3 1
b b b b b
| | | |
| |
= = =
| |

\ . \ .


Since 2 points on the line l lies on the plane H
3
, the line l must then lie on the plane
H
3
. Thus b = 0.

Alternative Method 2
2 1
0 4 0
2 1
b b b
| | | |
| |
= =
| |

\ . \ .
and
1 1
1 0 0
1 1
b b
| | | |
| |
= =
| |

\ . \ .


Since point P on line l lies on plane H
3
and line l is

parallel to H
3
, therefore line l
must then lie on the plane H
3
. Thus b = 0.

Alternative Method 3
If the 3 planes intersect at a common line, the common line of intersection between
any of the 2 planes must be the same.

For the line of intersection between
1
t and
2
t to be the same as the line of
intersection between
1
t and
3
t , P must lie on
3
t and the 2 lines must be parallel.
2 1
0 4 0
2 1
b b b
| | | |
| |
= =
| |

\ . \ .
and
1 1 1
3 1 for some
2 1 1
3 2 1
3 1
3 1
3 and 3 2 3 and 3 3
0
b
b
b
b b
b

| | | | | |
| | |
=
| | |

\ . \ . \ .
| | | |
| |
=
| |
+
\ . \ .
= = + =
=


Thus b = 0.

6(i)
Since
3
2
5
2
3
MN
| |
|
|
=
|
|
\ .
and
4
3
5
BH
| |
|
=
|
|
\ .

the acute angle between the lines
1
l and
2
l
3
2
5 9 25
2 1 1 2 2
3
2
5
2
3
4
3
5
12
cos cos 80.3
50 17.5
3
4
3
5

| |
| |
| | |
|
|
|
|
-
|
|
|
|
|
\ . | | |
+ +
| \ .
|
= = =
|
|
| |
| \ . | |
|
|
|
|
|
|
|
|
|
\ . |
|
\ .
\ .


6(ii) A vector normal to the plane EFGH is k

The acute angle between the plane EFGH and a line parallel to AP
1 1
12 0
1
21 0
10
35 1
3.5
sin sin 55.4
12 0 18.1
1
21 0
100
35 1

| |
| | | |
|
| |
-
|
| |
| |
|
| |
\ . \ .
= = = |
|
| | | |
| \ .
| |
|
| |
|
| |
|
\ . \ .
\ .


7(i) For the population to grow, population at the end of (n+1)th week must be greater
than the population at the end of nth week.
1
1
0
4 0
n n
n n
n n
u u
u u
u qu
+
+
>
>
>

Re-arranging,
1
4
4(1 )
1
n
u q
q

> =

.
Hence there must be initially
1
4(1 ) q

fruitflies in the jar.

7(ii) Given that
1
10 u = and
2
15 u = , we have
( )( ) 15 2 10 4 q =
which gives 0.1 q = .
The recurrence relation is
1
1.9 4
n n
u u
+
= .
Using the GC to evaluate the terms, we have
20
1100000 1000000 u = ~ (1 s.f.)
GC key strokes:
Press MODE, select seq at the 4
th
line. Press y=. Enter the following:
Press 2
nd
, GRAPH.

This is highly impractical as the formulation of the recurrence relation did not take
into account the physical constraints of the jar.

7(iii) The suggestion should not be taken up as the differential equation gives a continuous
model as an approximation which only works when the population is large enough.

8 Let P
n
be the statement
| | ( )
2
2
( 1)! 1 ! 2
n
r
r r n
=
= +

where , 2. n n e >

When 2 n = ,
LHS ( )
2
2 1! 4 = = , RHS 3! 2 6 2 4 = = =
Hence, P
2
is true.

Assume P
k
is true for some , 2. k k e >
| | ( )
2
2
i.e. ( 1)! 1 ! 2
k
r
r r k
=
= +

.
Want to prove that P
k+1
is true,
| | ( )
1
2
2
i.e. to prove ( 1)! 2 ! 2
k
r
r r k
+
=
= +

.
LHS = | | ( )
2 2
2
( 1)! ( 1) !
k
r
r r k k
=
+ +


( ) ( ) ( ) 1 ! 2 1 1 ! k k k = + + + + (



( )
( )
1 !(1 1) 2
2 ! 2
k k
k
= + + +
= +

RHS =

Hence P
k+1
is true whenever P
k
is true. Since P
2
is true, by using mathematical
induction, P
n
is true for all , 2. n n e >

Replacing ( 1) with r r +
( ) | |
| |
1
2 2
2 3
2 2
2
( 1) ! ( 1)!
( 1)! 2 (2 1)!
( 1)! 2 4
( 1)! 6
n n
r r
n
r
r r r r
r r
n
n

= =
=
+ =
=
= +
= +









Alternatively
Replacing with ( 1) r r +

| | ( )
( )
( ) | |
1
2 2
2 1
1
2 2
2
1
2 2
2 2
( 1)! ( 1) !
( 1) ! 2 (1)!
( 1) ! ( 1)! 2
( 1)! 4 2
( 1)! 6
n n
r r
n
r
n n
r r
r r r r
r r
r r r r
n
n

= =

= =
= +
= + +
+ =
= +
= +




9











(i)





(ii)

Equating the two equations, we have
4 2
3
0
4
+ = x x
Solving,
1
2
= x or
1
2
= x

1
2
= y or
1
2
= y
The coordinates of point A and B are
1 1
( , )
2 2
and
1 1
( , )
2 2
respectively.

Area of R = 2
1 3
2 2 2 2
1
0
2
3
d d
4
x x x x
| |
|
+
|
\ .
} }

= 2 ( 0.11785 + 0.05403)
= 0.34 (2 d.p.)

Volume =
1
2 2
0
3
d
4
y y y t
| |

|
\ .
}

=
1

3 2
2
0
3
4 3 2
y y
y t
(

(


=
5
24
t

10(i) b = 2
10(ii)
2
2 3 x ax a
y
x b
+ +
=
+


2 2
2 2
d (4 )( 2) (2 3 ) (2 8 5 )
d ( 2) ( 2)
y x a x x ax a x x a
x x x
+ + +
= =


For C to have no stationary points,
d
0 has no real roots.
d
y
x
=
2
i.e. 2 8 5 0 has no real roots
Discriminant = 64 4(2)( 5 ) 0
8
8 5 0 , so (Ans)
5
x x a
a
a a
=
<
+ < <


Alternative Method
For C to have no stationary points,
d
0
d
y
x
=
2
2
2
i.e. 2 8 5 0
2( 2) (8 5 ) 0
Since 2( 2) 0 for all ,
8
8 5 0 (Ans)
5
x x a
x a
x x
a a
=
+ =
> e
+ < <


10(iii)
Given that a = 3,
2
2 3 9 7
2 1
2 2
x x
y x
x x

= = +


















10(iv) Scale C parallel to the y-axis (with x-axis invariant) by a factor of 1/3, then translate
the resulting curve by 5 units in the positive y-direction to obtain the required curve.

Alternatively,
Translate C 15 units in the positive y-direction, then scale parallel to the y-axis (with
x-axis invariant) by a factor of 1/3 to obtain the required curve.
11(i)
( )
d 1
d 1
y
x x x

+ ( )
d
d 1
y k
x x x
=
+
for some real constant k.

Since
d 1
d 2
y
x
= when x = 2,
( )
1
1
2 2 1
k
k = =

.
x
y
0
y = 2x+1
x=2
(3, 0)

( )
d 1
d 1
y
x x x
=
+
(Shown)

d 1 1
d 1
y
x x x
=
+


1 1
d
1
y x
x x
| |
=
|
+
\ .
}
ln 1 ln x x C = + + or
1
ln
x
y C
x
+
= +

11(ii)
y = ln 3 when x =
1
4

1
1
4
ln3 ln ln 3 0
1
4
C | | C C

= + = + =


ln 1 ln y x x = + or
1
ln
x
y
x
+
=

11(iii)














12(a)
(i)
First term of first row 1 =
First term of second row 1 1 = +
First term of third row 1 1 2 = + +
First term of fourth row 1 1 2 3 = + + +
First term of
th
n row ( ) 1 1 2 3 ...... 1 n = + + + + +
( ) ( )
1
1 1 1
2
n
n

= + +
( )
1
1 1
2
n n = +

(ii)
Sum of the sum of all the elements
th
n row
( ) ( )
2 1
2
n
a d n = +
( ) ( )
1
2 1 1 1 1
2 2
n
n n n
| | | |
= + +
| |
\ . \ .

( )
2
1
1
2
n n = +


For
( )
2 5
1
1 10
2
n n + >
When 58, n = sum of elements 97585 =
When 59, n = sum of elements 102719 =
Least value of n is 59.

0
x

y = 1
y
x = 1


12(b)

4 8
16
17
S S =
( ) ( )
4 8
1 1
16
1 17 1
a r a r
r r

=


4 8
17 17 16 16 r r =
8 4
16 17 1 0 r r + =
From GC,
1
2
r =
Sum to infinity
1
2
2
1
a
a = =

.




13(i) y = f(2x 3)











13(ii)















13(iii)












x
y




0
y
x


y = f (x)
0


x
y
(0,0)






14(i)
(a)
Complex No
(b)


14(ii)



END

2009 RI(JC) H2 Maths Preliminary Examination Paper 2 (Solutions)

Section A: Pure Mathematics [40 marks]

1
(i)
(ii)
(iii)
Complex No
1(a)
1(b)
2(i)
| ) | ) 0, 2,
f f
R D = _ = .
Hence the composite function ff may be defined.
ff : 2 2, 2 x x x + + > and the range of ff is
)
2,

.

(ii)


(iii) v = f(v).
From the graph in part (b), the graph of y = f(x) intersects the graph of y = x at x = 2.
Hence, v = 2.

(iv) Range of f
n
= {2}.

3(i)
Cost,
( ) ( )
2
0.09 2 0.06 2 C r rh t t = +

2
2
2
300
0.18 0.12
36
0.18
r r
r
r
r
t t
t
t
| |
= +
|
\ .
= +

2
3
3
2
3
2 3
d 36
0.36 0
d
100
100
d 72 100
0.36 0 when
d
C
r
r r
r
r
C
r
r r
t
t
t
t
t
= =
=
=
= + > =

The cost is minimized when
3
100
r
t
= .
So a = 100

(ii) Let V and H be the volume and height of the liquid respectively.
When t = 0,
2
500 (2 ) h h t t =

3
125
5
h
h
=
=

2
(2 ) V h H t =
100 H t =
d d
100
d d
V H
t t
t =

100 ( 0.02)
2
t
t
=
=


The volume of the liquid is decreasing at
3 -1
2 cm s t 2 minutes after the start of the
experiment.

4(i) A normal vector to plane is
y = f(x)
y = ff(x)
(2, 2)
y = x
2


1 3 2
2 0 1
0 1 6
| | | | | |
| | |
=
| | |

\ . \ . \ .

Equation of plane is
2 2 2
1 3 1
6 2 6
2
1 5
6
| | | | | |
| | |
=
| | |

\ . \ . \ .
| |
|
=
|
\ .
r
r

(ii)
0 2
: 2 1 ,
0 6
AB
l
| | | |
| |
= + e
| |
\ . \ .
r
Thus
0 2
2 1 for some
0 6
OB
| | | |
| |
= +
| |
\ . \ .
.
Since B lies on the plane:
0 2 2
2 1 1 5 4 (2 ) 36 5
0 6 6
7

41

( | | | | | |
( | | |
+ = + + + =
( | | |
(
\ . \ . \ .

=


Thus
14
1
75
41
42
OB
| |
|
=
|

\ .
.
14 0 14
1 1
75 2 7
41 41
42 0 42
AB
| | | | | |
| | |
= =
| | |

\ . \ . \ .


Perpendicular distance from A to H =
1 7 41
2009
41 41
= units.

(iii)
Length of projection of OA on H
=
0 2 12
1 1 10
2 1 0 4
41 41 41
0 6 4
| | | | | |
| | |
= =
| | |

\ . \ . \ .


(iv)
Hence area of OAB A
2
1 7 41 10
4
2 41 41
14 10
units
41
| || |
=
| |
| |
\ .\ .
=


Section B: Statistics [60 marks]


5(i)

Number of arrangements = ( ) 4 1 ! 4!
= 144.

(ii)

Number of arrangements = ( ) 7 1 ! 2! 8

= 11520.

6(a)
(i)

Since A and B are mutually exclusive,
( ) ( ) ( ) P P P A B A B = +
( ) ( ) ( ) P P P B A B A = = 0.6 0.2 = 0.4



6(a)
(ii)

Since A and B are independent,
( ) ( ) ( ) P P P A B A B = = ( ) 0.2 P B (1)
Also, ( ) ( ) ( ) ( ) P P P P A B A B A B = +
( ) ( ) ( ) ( ) P P P P B A B A A B = +
( ) ( ) P 0.6 0.2 0.2P B B = + [from (1)]
( ) 0.8P 0.4 B = ( ) P 0.5 B =


6(b)
(i)

Required probability
= (0.1 0.01) (0.6 0.15) (0.3 0.25) + +
= 0.166


6(b)
(ii)

( )
( )
( )
P good risks and no accidents
P good risks | no accidents
P no accidents
=
( ) P good risks | no acciden
0.1 (1 0.01)
1
ts
0.166

=

[from (i)]
( ) P good risks | no accide
0.099 33
0.834
n
78
ts
2
= =
or 0.119 (3 s.f.)


7(a)


Let Y be the number of observations of X that are less than 1, out of 88. Then Y ~ B(88,
0.04).
Since n = 88 > 50 is large and p = 0.04 < 0.1 is small, is such that np = 3.52 < 5, Y ~
Po(3.52) approximately.
Required probability = P(Y > 6) = 1 P(Y s 5)
= 1 0.85496 or 0.14504 (5 s.f.)
= 0.145 (3 s.f.)


7(b)
(i)


Let W be the number of larvae in 5ml of pond water.
Then W ~ Po(4 0.5), i.e. W ~ Po(2).
P(W < 3) = P(W s 2) = 0.67668 (5 s.f.)
= 0.677 (3 s.f.)


7(b)
(ii)


Let T be the total number of larvae in 6 test-tubes, with each filled with 5ml of pond
water.
Then T ~ Po(6 2), i.e. T ~ Po(12).
Since = 12 > 10 is large, T ~ N(12, 12) approximately.
P(T > 13) = P(T > 13.5) (by continuity correction)
= 0.33250 (5 s.f.)
= 0.333 (3 s.f.)




8
(i)
Given X ~ N(20,
2
o ).
P(12 ) P( ) 0.42065 X k X k < < = > =
P( 12) 1 2(0.42065) 0.1587 X s = = .

12 20
P( ) 0.1587 Z
o

s = where Z ~ N(0, 1)

8
0.99982
o

= (5 s.f.) (from graphic calculator)



8
8.0015
0.99982
o

= =

(5 s.f.)

Hence, o = 8. (nearest whole number) (shown)

P( ) 0.42065 X k > =
P( ) 1 0.42065 0.57935 X k s = =
k = 21.6. (3 s.f.) (from graphic calculator)



8
(ii)

Using o = 8, X ~ N(20,
2
8 ).

1 2
4 X Y Y = +

E( ) X =
1 2
E( 4) Y Y +

20 = 2 E( ) 4 Y
E( ) Y =
20 4
2
+
= 12.

Also, Var( ) X =
1 2
Var( 4) Y Y +

64 = 2 Var( ) Y (since
1
Y and
2
Y are independent)

Var( ) Y = 32.

Y ~ N
( )
( )
2
12, 32 .

E(W 2Y ) = E(W) 2E(Y) = 25 2(12) = 1.
Var(W 2Y ) = Var(W) + 2
2
Var(Y) = 16 + 4(32) = 12
2
.

(W 2Y ) ~ N(1, 12
2
).
P( 2 ) P( 2 0) W Y W Y > = > = 0.533. (3 s.f.)


9
(i)

Select a sample of 50 students that is proportional to the size of each CCA type as
shown in the table:





Sports


Clubs

Performing
Arts

Uniform
Groups

48
50 16
150
=


60
50 20
150
=


33
50 11
150
=


9
50 3
150
=


Within each stratum, use simple random sampling or systematic random sampling.


9
(ii)

Since the objective of the survey is to gather feedback on CCA experiences, using
stratified random sampling ensures that all 4 CCA types are fairly represented in the
sample, whereas simple random sampling does not ensure that.

9
(iii)


Let y = x 5. Then 32 y =

and
2
400 y =

.
1 32
0.64
50
y y
n
= = =

.
( ) ( )
2 2
2 2
1 1 1 1
400 32
1 49 50
379.52 9488
7.7453. (5 s.f.)
49 1225
y
s y y
n n
( (
= =
( (


= = =



Let x and
2
x
s be the unbiased estimates of the required mean and variance respectively.

5 0.64 5 5.64 x y = + = + = .
2 2
9488
7.75
1225
x y
s s = = = . (3 s.f.)


9
(iv)


Since n = 50 is large, by Central Limit Theorem,
7.7453
~ N(5.64, )
50
X approximately.
i.e. ~ N(5.64, 0.15491) X approximately.
P( X > 6) = 0.180. (3 s.f.)

10
(i)

Let X be the time taken to heat 2 litres of water from 25
O
C to 100
O
C, in seconds.
Test: H
0
: = 440 vs H
1
: < 440, where = E(X).

Perform a one-tail test at the 5% level of significance.

Under H
0
,
T =
0
/
X
S n

~ t(n1)

where
0
= 440, x = 428.92, s = 20.602 and n = 12.

Using a t-test, p-value = 0.0446 (3 s.f.)
Since p-value = 0.0446 < 0.05, we reject H
0
and conclude that there is sufficient
evidence, at the 5% level of significance, that the manufacturers claim is valid.

10
(ii)

The z-test is the correct test because X follows a normal distribution and Var(X) is
known.

10
(iii)

Since the correct test (z-test) does not reject the null hypothesis, then p-value (for
correct test) > 0.05.

Also, from the graphs below,










we see that

p-value (for t-test) > p-value (for z-test)
p-value (incorrect test) > p-value (correct test) > 0.05



p-value
(z-test)

Distribution
of Z ~ N(0, 1)

0
0
Distribution
of T ~ t(11)




p-value
(t-test)

p-value (incorrect test) > 0.05
Hence, we always do not reject the null hypothesis using the incorrect test, i.e. it is not
possible for the incorrect test (t-test) to reject the null hypothesis.


11
(i)

From the graphic calculator,
the least square regression line of y on x is
y = 1.17088 + 9.7296x.
the least square regression line of x on y is

x = 0.352 + 0.0927y. (3 s.f.)
and the product moment correlation coefficient for the data is 0.950. (3 s.f.)


11
(ii)











From the graphic calculator,
the scatter plot of y against x is as shown above.
The required coordinates is ( x , y ), i.e. (4.7, 46.9).


11
(iii)

When x = 12, y = 1.17088 + 9.7296(12) = 117.92608.
Hence, required predicted score = 117.92608 marks.
The estimate is not reliable as x = 12 is not within the domain of the given data set.
[OR the scatter plot in (ii) does not suggest a linear relationship between x and y.]
[OR the predicted score of 117.92608 > 100 marks]

11
(iv)

From the graphic calculator,
the least square regression line of y on
1
x
is
y = 101
198
x
. (3 s.f.)
and when x = 12, y = 101.498
198.46
12
= 85.0. (3 s.f.)

Hence, required predicted score = 85.0 marks. (3 s.f.)
The scatter plot of y against
1
x
below suggests a linear relation between of y and
1
x
,
whereas that of y against x in (ii) does not.


[OR as x , y in (iii), but y 101 in (iv), which
means that for large values of x, it is more realistic to
use regression line in (iv) to estimate y.]
Thus, this is a better estimate than that found in (iii).




(4.7, 46.9)













x = 0.352 + 0.0927y



x

0



10



100


y








y = 1.17 + 9.73x
2009 TJC Preliminary Examination
H2 Mathematics 9740 Paper 1 Solutions

Question 1
Find the real numbers a and b such that
2
2
5 3 1
( 1)( 2)
x x
x x
+
+
=
2
1
1 2
ax b
x x
+
+
+
. [2]
Hence, find
2
2
5 3 1
( 1)( 2)
x x
x x
+
+
}
dx. [3]
Solution

2
2
5 3 1
( 1)( 2)
x x
x x
+
+
=
2
1
( 1) ( 2)
ax b
x x
+
+
+

2
5 3 1 x x +
2
( 1)( 2) ( 1) ax x b x = + + +

Comparing constant term: 1 2 b = + b = 3
Comparing coefficients of
2
x : 5 = a + b a = 2

2
2
5 3 1
( 1)( 2)
x x
x x
+

+
=
2
2 1 3
( 1) ( 2)
x
x x
+
+
+
.


2
2
5 3 1
( 1)( 2)
x x
x x
+
+
}
dx =
2 2
2 1 3
d d d
1 1 2
x
x x x
x x x
+ +
+ +
} } }

=
2 1
ln( 1) tan 3ln 2 x x x C

+ + + +

Question 2
Without the use of a calculator, solve the inequality
2
3
2
6 11
x
x x
>
+
. [5]
Hence, find the range of values of x which satisfies
2
3
1
4 12 11
x
x x
e
e e
>
+
. [2]
Solution


2
3
2
6 11
x
x x
>
+
--------------(*)
Since
2 2
6 11 ( 3) 2 0 x x x + = + > for all real x,

2
3 2( 6 11) x x x > +

2
2 15 22 0 x x + <
( )( ) 2 2 11 0 x x <

11
2
2
x < <

Alternative Method:

2
3
2
6 11
x
x x
>
+
--------------(*)

2
2
3 2( 6 11)
0
6 11
x x x
x x
+
>
+


2
2
2 15 22
0
6 11
x x
x x
+
<
+

( )( )
2
2 2 11
0
6 11
x x
x x

<
+

Since
2 2
6 11 ( 3) 2 0 x x x + = + > for all real x,
( )( ) 2 2 11 0 x x <
11
2
2
x < <

2
3
1
4 12 11
x
x x
e
e e
>
+

( ) ( )
2
3(2 )
2
2 6 2 11
x
x x
e
e e
>
+

Replace x by 2
x
e in (*),
11
2 2
2
x
e < <
11
1
4
x
e < <
11
0 ln
4
x < <


Question 3
The sequence u
1
, u
2
, u
3
, is defined by ( )
1
!
n
n
r
u r r
=
=

for all positive integers n.


(i) Calculate the values of 1
n
u + for n =1, 2, 3 and 4. [2]
(ii) Make a conjecture for
n
u in terms of n. [1]
(iii) Prove your conjecture in part (ii) by mathematical induction. [4]

Solution
(i) ( )
1
1 1 1 1! 2 u + = + =
( ) ( )
2
1 1 1 1! 2 2! 6 u + = + + =
( )
3
1 6 3 3! 24 u + = + =
( )
4
1 24 4 4! 120 u + = + =
(ii) Observe from part (i), ( ) ( ) 1 1 ! 1 ! 1
n n
u n u n + = + = +
(iii) Let P
n
be the statement ( ) 1 ! 1
n
u n = + for all n
+
e .
When n = 1, LHS = ( )
1
1
1 1! 1
r=
=

and RHS = ( ) 1 1 ! 1 1 + =

1
P is true.
Assume P
k
is true for some k
+
e , i.e. ( ) 1 ! 1
k
u k = + .
When 1, n k = +
( ) ( ) ( )( )
1
1
1 1
! ! 1 1 !
k k
k
r r
u r r r r k k
+
+
= =
= = + + +


= ( ) ( )( ) 1 ! 1 1 1 ! k k k + + + +
= ( ) ( ) 1 ! 1 1 1 k k + + +
= ( ) 2 ! 1 k +

1
P is true P is true
k k+

Since
1
P is true and
1
P is true P is true
k k+
, by Mathematical Induction,
P
n
is true, i.e. ( ) 1 ! 1
n
u n = + for all n
+
e


Question 4
A radioactive substance decays at a rate which is proportional to the square of the mass remaining. Initially,
the mass is 100 mg and after 9 days, the mass remaining is 25 mg. Letting x be the mass of radioactive
substance remaining and t be the time in days, find x in terms of t. [5]
Determine the total time (to the nearest day) after which the mass remaining is 7 mg. [2]

Solution

2
d
d
kx
t
x
=
t k
x
x
d 1
d
2 } }
=
c kt
x
+ =
1

When t = 0, x = 100
100
1
= c

When t = 9, x = 25
300
1
100
1
9
25
1
= + = k k

3
300
100
1
300
1
+
=
+
=
t
t
x
When x = 7, 857 . 39 857 . 42 3
3
300
7 = = +
+
= t t
t


Total time needed for mass to reach 7 mg is 40 days.
Question 5
(a) Find the roots of the equation
4
1 z i = + in the form re
iu
where r e and t < u s t.
[5]
(b) In an Argand diagram, O represents the origin and P represents the complex number
4 + 3i. Given that O, P, Q and R are the vertices of a square described in an
anti-clockwise orientation, find the complex numbers represented by Q and R. [4]

Solution
(a)
4
4
1 2
i
z i e
t
= + =

( 2 )
4
2
i k
e
t
t +
=

1
16 2
8
2 , 0, 1, 2
k
i
z e k
t t | |
+
|
\ .
= =

1 1 9 1 7 1 15
8 16 8 16 8 16 8 16
2 , 2 , 2 , 2
i i i i
z e e e e
t t t t

=


(b) Let P, Q and R be the points on the Argand diagram representing the complex numbers p, q and r
respectively.

Method 1
Since OPQR is a square, r = ip
and (4 3 ) ( 3 4 ) 1 7 q p r i i i = + = + + + = +
(4 3 ) ( 3 4 ) 1 7 q p r i i i = + = + + + = + [M1, A1]

Method 2: By sketching using Geometry














Note:
It is mathematically incorrect to write the following;
1. OR iOP = or OR iOP =
2. 3 4 OR i = + or 3 4 OR i = +

You should define the complex numbers in your workings as shown in the solution.



Re(z)
Im(z)
P (4, 3)
Q (1, 7)
R (-3, 4)
O
Question 6
Referred to the origin O, the position vectors of A and B are i + 3j + 2k and 3i + 2j k respectively.
(i) Find the position vector of M, where M is the mid-point of A and B. [2]
(ii) Find the cosine of angle AOB. [2]
(iii) OM is extended such that OX = 2OM. Using the result in (ii), find the exact area of OAXB. What is the
geometrical shape of OAXB? [4]

Solution

(i) Method 1: Using Ratio Theorem
OP =
2
1
( OA + OB )
1 3
1
3 2
2
2 1
( | | | |
( | |
= +
( | |
| |
(

\ . \ .


4 2
1
5 2.5
2
1 0.5
| | | |
| |
= =
| |
| |
\ . \ .


Method 2: Law of Vector Addition
2
1
3
AB OB OA
| |
|
= =
|
|

\ .

1 2 2
1 1
3 1 2.5
2 2
2 3 0.5
OM OA AM OA AB
| | | | | |
| | |
= + = + = + =
| | |
| | |

\ . \ . \ .


(ii)
1 3
3 2
2 1
3 6 2 1
cos
14 2 | || | 14 14
.
OA OB
AOB
OA OB
| | | |
| |
| |
| |

+
\ . \ .
= = = =

(iii) Area of OAXB =
1
2 ( )( ) sin
2
OA OB AOB
=
3
14 14 147 7 3
2
| |
= =
|
|
\ .
units
2


OAXB is a rhombus since OA = OB and 90 AOB = .









O
A B M
Question 7
A sequence of positive real numbers
0
x , x
1
, x
2
, x
3
, ... satisfies the recurrence relation
1
1
2 1
3
n
n
n
x
x
x

+
=
+
, for positive integers n.
(i) As n ,
n
x o . Determine the exact value of o . [3]
(ii) Describe the behaviour of the sequence when
0
0.5 x = . [1]
(iii) Show that
1 n n
x x
+
< when
n
x o > . [4]

Solution
(i) As n ,
n
x o ,
2 1
3
o
o
o
+
=
+


2
3 2 1 o o o + = +

2
1 0 o o + =
o =
1 5
2
+
or
1 5
2

(rejected
n
x > 0)

(ii) Using GC, the sequence is increasing and converges to o when
0
0.5 x = .
(iii) Method 1 (Graphical method)

1
2 1
3
n
n n n
n
x
x x x
x
+
+
=
+

Sketch the graph of
2 1
3
x
y x
x
+
=
+
and
observe that when x > o,
2 1
0
3
x
x
x
+
<
+

Therefore, replace x by
n
x ,
1 1
2 1
0 0
3
n
n n n n n
n
x
x x x x x
x
+ +
+
< < <
+











Another possible graphical method is to check that
2 1
3
x
x
x
+
<
+
when x > o:
Then need to sketch graphs of
2 1
3
x
y
x
+
=
+
and y x = on the same diagram as shown below


Method 2
1
2 1
3
n
n n n
n
x
x x x
x
+
+
=
+

=
2
2 1 3
3
n n n
n
x x x
x
+
+

=
( )
2
1
3
n n
n
x x
x
+
+

=
( )
1 5
2
3
n n
n
x x
x
o
| |
+
+
|
\ .
+
from part (i)
Since 0
n
x > ,
1 5
2
n
x
+
+ >0, 3
n
x + > 0.
Also
n
x o > 0
n
x o >

1 1
0
n n n n
x x x x
+ +
< <

Alternative to explain why
( )
2
1
0
3
n n
n
x x
x
+
<
+
:
Sketch the graph of y =
2
1 x x +









Since
n
x o > ,
2
1
n n
x x + > 0. Also, 3
n
x + > 0.
1 1
0
n n n n
x x x x
+ +
< <


1 5
2

o
Question 8
The functions f and g are defined by
( )
2
f : 2 4, x x + , 2 e s x x ,

( )
2
1
g : ,
7 4
x
x
xe , x s k, x 5, x 9.

(a) (i) Sketch the graphs of f and f
1
on the same axes, showing the relationship
between the two graphs. [3]
(ii) If ( ) ( )
1
f f o o

= , show that
2
3 0 o o = . [1]
(b) State the maximum value of k such that the inverse of g exists. Hence find [1]
(i) the inverse of g, stating its domain, [3]
(ii) the range of gf. [1]


Solution:

(a) (i)










(ii) If ( ) ( )
1
f f o o

=
( ) f o o =
( )
2
2 4 o o + =

( )
2
4 4 4 o o o + + =

2
2
4 4 4 0
3 0
o o o
o o
+ + =
+ =


2
3 0 o o =

(b) Maximum value of k = 7

(i) Let
2
1
( 7) 4
y
x
=



2
1
( 7) 4 x
y
=
( )
2 1
7 4 x
y
= +
1
7 4 x
y
= +
( )
1 1
7 4 or 7 4 rejected 7 x
y y
= + + + s
y =f
1
(x)
y=f(x)
y = x
(2, 4)
(4, 2)

1
1 1
g : 7 4 , , , 0
4
x x x x
x

+ e s >
or
1
g
1
D ( , ] (0, )
4

=

(ii)
gf
1
R 0,
5
| (
=

(
\

Method 1 By mapping, (0, 2] f (, 4] g
1
0,
5
| (

(
\
R
gf
=
1
0,
5
| (

(
\
.
Method 2 Using the graph of gf with (, 2] as domain to get R
gf
=
1
0,
5
| (

(
\
.
graph of gf :





Question 9
(a) Find the sum of the arithmetic series
(m + 1) + (m + 3) + (m + 5) + . . . . + (3m 3)
where m is a positive integer. [4]

(b) A customer purchases a new 52-inch LCD television set from Counts Hypermarket for $7000 and
decides to pay the entire amount by loan instalment. He takes the loan at the beginning of June during
the Mid-Year Sale and he repays $p at the end of each month where p < 7000. The Hypermarket will
then charge a 5% interest on the outstanding balance after each monthly repayment. Show that the
outstanding amount owes at the end of the nth month is
( )
7000(1.05) 21 1.05 1
n n
p . [3]
Deduce the least monthly repayment amount p, rounded off to the nearest dollar, required to pay off
the entire loan by the end of the 12
th
month. [3]

Solution
(a) Let n be the number of terms.
3m 3 = m + 1 + (n 1)2
n = m 1
Sum =
| |
1
1 (3 3)
2
m
m m

+ +
=
1
( 1)(4 2)
2
m m
=(m 1)(2m 1)


(b) month Start of month End of month
1 7000 (7000 p)1.05

2 (7000 p)1.05 [(7000p)1.05 p]1.05
=(7000)1.05
2
p(1.05
2
+ 1.05)

3 (7000)1.05
2
p(1.05
2
+ 1.05) [(7000)1.05
2
p(1.05
2
+ 1.05) p]1.05
=(7000)1.05
3
p(1.05
3
+ 1.05
2
+ 1.05)

At the end of the nth month, the outstanding amount owes
=(7000)1.05
n
p(1.05
n
+ 1.05
n1
+ . . . + 1.05)
=
( )
1.05 1.05 1
7000(1.05)
1.05 1
n
n
p


=
( )
7000(1.05) 21 1.05 1
n n
p (shown)

To pay off the entire loan by the 12
th
month, the outstanding amount owes at the end of the 12
th
month must
be less than or equal to 0.

( )
12 12
7000(1.05) 21 1.05 1 p s 0

12
12
7000(1.05 )
(1.05 1)
21
p >
752.17 p >

The least monthly instalment is $753.



Alternative:
To pay off the entire loan by the 12
th
month, the outstanding amount owes at the end of the 11
st
month must
be less than or equal to p.

( )
11 11
7000(1.05) 21 1.05 1 p p s
p[ 1 + 21(1.05
11
1) ] >7000(1.05)
11

p > 752.17

The monthly instalment is $753.

Question 10
Let ( ) tan ln(1 ) y x = + .
Prove that
2
d
(1 ) 1
d
y
x y
x
+ = + . [2]
Find Maclaurins series for y, up to and including the term in
3
x . [6]

Hence, find the Maclaurins series expansion of y = sec
2
( ) ln(1 ) x + up to and including the term in x
2
. [2]

Solution
Given ( ) tan ln(1 ) y x = + ,
1
tan ln(1 ) y x

= + ,
Differentiating both sides of the equation wrt x, we obtain
2
1 d 1
1 d 1
y
y x x
=
+ +
,
2
d
(1 ) 1
d
y
x y
x
+ = +
Alternative:
2
d 1
sec (ln(1 )).
d 1
y
x
x x
= +
+

=
2
1
(1 tan (ln(1 )).
1
x
x
+ +
+

=
2
1
(1 ).
1
y
x
+
+


2
d
(1 ) 1
d
y
x y
x
+ = +
Differentiating both sides wrt x,

2
2
d d d
(1 ) 2
d d d
y y y
x y
x x x
+ + = ,
2
2
d d
(1 ) (1 2 ) 0
d d
y y
x y
x x
+ + = .
Differentiating both sides again wrt x,

2
3 2 2
3 2 2
d d d d
(1 ) (1 2 ) 2 0
d d d d
y y y y
x y
x x x x
| |
+ + + =
|
\ .
,

2
3 2
3 2
d d d
(1 ) (2 2 ) 2 0
d d d
y y y
x y
x x x
| |
+ + =
|
\ .
.
When x = 0, y = 0,
d
1
d
y
x
= ,
2
2
d
1
d
y
x
= ,
3
3
d
4
d
y
x
= .


Hence,
2 3 2 3
4 2
... ...
2! 3! 2 3
x x x x
y x x = + + = + +
| | | |
2 2
sec ln(1 ) 1 tan ln(1 ) x x + = + +
=
2
1 y +
= 1+
2 3 2 3
2 2
( )( )
2 3 2 3
x x x x
x x + +
=
2
1 .... x + +

Alternative: Differentiating both sides wrt x,

2 2
1
sec (ln(1 )). 1 2 ...
1
x x x
x
+ = + +
+


2 2
sec (ln(1 )) (1 )(1 2 ) x x x x + = + +
=
2 2
1 2 ... x x x x + + +
=
2
1 ... x + +

Question 11

The region R shown in the figure below is bounded by the graph of 2 y x = and the ellipse
2
2
1
4
y
x + = .
(i) Express the area of R in the form

0
f ( ) d
a
x x b +
}
where a and b are constants to be determined
and f(x) is a function of x. Hence find the exact area of R by means of the substitution x = sin
u .

(ii) Find the volume of the solid generated when R is rotated t radians about the y-axis.














Solution










[7]

[3]
(i)
2
2 2 2 2
1 4(1 ) 2 1
4
y
x y x y x + = = =
The graphs intersect at
2
, 2
2
x y = =
Area of R
=
2
2
2
0
1 2
2 2 1 d ( 2)
2 2
x x
( | |

( |
|
(
\ .
}



O
x
y


1
2
2
1
R
2
2
2
0
4 1 d 1 x x =
}

=
2
4
0
4 1 sin cos d 1
t
u u u
}

2
4
0
2 2cos d 1
t
u u =
}

( )
4
0
2 1 cos 2 d 1
t
u u = +
}

| |
4
0
2 sin 2 1
t
u u = +
= 1 1
2 2
t t | |
+ =
|
\ .


(ii) Required volume =
( )
2
2
2
2
1 2
2 1 d
3 2 4
y
y t t
| |
| |
+
|
|
|
\ .
\ .
}

= 1.23

Question 12

The lines l and m have the equations
r =
|
|
|
.
|

\
|
+
|
|
|
.
|

\
|

1
1
1
2
4
15
and r =
2 1
7 3
3 2

| | | |
| |
+
| |
| |
\ . \ .

respectively. The plane H has the equation r .
2
0 20
1
| |
|
=
|
|

\ .
.
(i) Find the position vector of the point of intersection between l and m.

(ii) Find the position vector of the point of intersection, A, of l and H.

(iii) Show that m and H have no common point.

(iv) Find, in the form r.n = p, an equation of the plane containing m and A.







[3]

[3]

[2]

[4]
Solution

(i)
|
|
|
.
|

\
|
+

+
=
|
|
|
.
|

\
|
+


2 3
3 7
2
2
4
15

8 , 21
5 2
3 3
13
= =

=
= +
=





Position vector of pt of intersection =
|
|
|
.
|

\
|

19
17
6






(ii)
15 2
4 0 20
2 1
.

+ | | | |
| |
=
| |
| |
+
\ . \ .

48 20 2 2 30 = = + +
OA=
|
|
|
.
|

\
|
=
|
|
|
.
|

\
|
+

+
46
44
33
48 2
48 4
48 15

(iii)
1 2
3 0 0 //
2 1
. m
| | | |
| |
=
| |
| |

\ . \ .


2 2
7 0 7 20
3 1
.
| | | |
| |
= =
| |
| |

\ . \ .
a point on m does not lie on H.
m does not intersect H.

Alternative:
2 2
7 3 0 20
3 2 1
4 2 3 2 20 (no solution)
.


+ | | | |
| |
=
| |
| |
+
\ . \ .
+ =

Every point on m does not lie in H. Thus m does not intersect H.

(iv) A vector // to H is
|
|
|
.
|

\
|

=
|
|
|
.
|

\
|

|
|
|
.
|

\
|
43
51
35
46
44
33
3
7
2

A normal is
|
|
|
.
|

\
|
=
|
|
|
.
|

\
|
=
|
|
|
.
|

\
|

|
|
|
.
|

\
|

2
1
1
27
54
27
27
2
3
1
x
43
51
35

equation of plane required is
r.
1 2 1
1 7 1
2 3 2
.
| | | | | |
| | |
=
| | |
| | |
\ . \ . \ .
i.e. r. 15
2
1
1
=
|
|
|
.
|

\
|


2009 TJC Prelim Paper 2 Solutions
Section A: Pure Mathematics [40 marks]

1 Simplify
1 1
.
( 2) ( 2)( 4) k k k k

+ + +
Hence find the sum of the following series

1 1 1 1
.....
(1)(3)(5) (2)(4)(6) (3)(5)(7) (2005)(2007)(2009)
+ + + + ,

giving your answer to 3 decimal places. [5]


[Solution]
1 1 4 4
( 2) ( 2)( 4) ( 2)( 4) ( 2)( 4)
k k
k k k k k k k k k k
+
= =
+ + + + + + +


1 1 1 1
.....
(1)(3)(5) (2)(4)(6) (3)(5)(7) (2005)(2007)(2009)
+ + + +
2005
1
1
( 2)( 4)
k
k k k
=
=
+ +


2005
1
1 1 1
4 ( 2) ( 2)( 4)
k
k k k k
=
(
=
(
+ + +



1 1 1
4 1(3) 3(5)


1 1
2(4) 4(6)
+
1 1
3(5) 5(7)
+
+ .
.
.
1 1
2003(2005) 2005(2007)
1 1
2004(2006) 2006(2008)
1 1
2005(2007) 2007(2009)
+
+
(
+
(



1 1 1 1 1
4 3 8 2006(2008) 2007(2009)
(
= +
(


= 0.115 (3dp)
2 The equation of a curve is
c
y ax b
x d
= + +
+
where a, b, c and d are non-zero constants. The two
asymptotes of the curve intersect at the point ( ) 1, 4 and the curve

has a minimum point at ( ) 2,10 . Find the values of a, b, c and d. [5]


[Solution]
From the given equation, we can observe that the asymptotes are:
y ax b = + and x d =

Since the two asymptotes intersect at the point (1, 4),
1 d = and
4 a b + = --------- (1)


( )
2
d
d
1
y c
a
x
x
=


For the minimum point at (2, 10),
d
0
d
y
x
=

( )
2
0
2 1
c
a =


a c = --------- (2)


At the point (2, 10),
2 10 a b c + + = --------- (3)

Substitute (2) into (3), we get 3 10 a b + = --------- (4)

(4) (1), 3, 1, and 3 a b c = = =


3 A curve C has parametric equations

x =
2
t , y = ln (t + 5), for 5, t t > e .

(i) Find
d
d
y
x
in terms of t and hence find the range of values of t when the gradient of the curve is
positive. [3]

(ii) Region S is bounded by the curve C, the line 2ln3 y = and both axes.
Sketch the curve C and indicate region S. [1]
Express the area of S in the form ( ) f d
b
a
t t
}
and hence find the exact area of S. [5]

[Solution]
(i) Given ( )
2
, ln 5 , 5, x t y t t t = = + > e


d
2
d
x
t
t
= and
d 1
d 5
y
t t
=
+


d
d 1
d
d
d 2 ( 5)
d
y
y
t
x
x t t
t
= =
+


For
d 1
0
d 2 ( 5)
y
x t t
= >
+

( ) 5 0 t t + >
( ) 0 or 5 rejected 5 t t t > < >
0 t >


(ii) Area of S
2ln3
0
d x y =
}

4
2
4
1
dt
5
t
t

=
+
}


4
4
25
5 dt
5
t
t

= +
+
}

4
2
4
1
5 25ln 5
2
t t t

(
= + +
(


50ln3 40 = units
2

y
x
16
ln5
2ln3
x = t
2

y = ln(t + 5)
0 t = - 4
t = 4
4 In an Argand diagram, the point P represents the complex number z where


( ) ( )
2 cos 1 1 2sin z i = u+ + u
(i) Show that the locus of P is a circle with radius

2 and centre (1,1). [3]
(ii) Sketch the locus of P. [1]
(iii) Find the exact value of the minimum value of

z + 6 4i . [2]
(iv) Find the maximum value of ( ) arg 6 4 z i + . [3]


[Solution]
(i)
( ) ( )
2 cos 1 1 2sin z i u u = + +
( ) ( ) 1 2 cos sin z i i u u =
( ) 1 2 z i = since cos sin 1 i u u =
The locus of P is a circle with radius 2 and centre (1, -1). (shown)

Method 2
Let z x iy = + ,
( ) ( )
2 cos 1 1 2sin x iy i u u + = + +
Compare real and imaginary parts, we get
2 cos 1 x u = + and 1 2sin y u = +

( ) 1
1
cos and sin
2 2
y
x
u u
+

= =
Since
2 2
cos sin 1 u u + = ,
( ) ( )
( ) ( )
2 2
2 2
1 1
1 1 1 2
2 2
x y
x y
+
+ = + + =
The locus of P is a circle with radius 2 and centre (1, -1). (shown)


(ii)












(iii) Min. value of 6 4 z i +
( ) ( ) ( )
2 2
6 1 4 1 2 74 2 = + = units


Im(z)
Re(z)
(1, -1)
(iv)
2
sin 0.16515 rad
74
o o = =

5
tan 0.62025 rad
7
| | = =
( ) ( ) Max. arg 6 4 0.455 rad z i | o + = ~ (3sf)















Im(z)
Re(z)
(1, -1)


(-6, 4)
5 (a) A circle has two fixed points A and B such that AB is the diameter of length 10 cm.
A point P moves on the circle. Given that the length AP decreases at the rate of 0.4 cm s
-1
, find
the exact rate at which the angle PAB is changing when AP = 5 cm. [5]

[Solution]
(a)







Let AP = x and PAB u Z = .
Given 4 . 0 =
dt
dx
cm s
-1
, find
dt
du
.


1
cos cos
10 10
x x
u u

| |
= =
|
\ .


2 2
2
1 1
d 1
10 10
1
d
100
100
1
10
10
x
x
x
x
u


= = =

| |

|
\ .


At 5 cm x = ,
( )
-1
2
d d d 1 2 2 3
0.4 rad s
dt d d 75 25 3
100 5
x
x t
u u
= = = =




Method 2
Let AP = x and PAB u Z = .
Given 4 . 0 =
dt
dx
cm s
-1
, find
dt
du
.


10
cos
x
= u
Differentiate wrt t,

dt
dx
dt
d
10
1
) sin ( =
u
u

When x = 5,

60
2
1
cos = = u u

0.04 2 2 3
sin60 75 25 3
d
dt
u
= = =

rad s
-1

P
B
A
10
5 (b) The diagram below shows the graph of ( ) f y x = which has a minimum point at (0, 1) and
asymptotes y = 3 and 2 x = .

Sketch, on separate diagrams, the graphs of
(i) ( ) f
'
y x = , [3]
(ii) ( )
2
f y x = . [3]

Another curve ( )
2
2 2
1 x y m + = where m is a positive real constant is drawn on the same diagram as
the graph in part (ii). State the range of values of m such that there are at least 5 points of intersection
between the two curves. [1]

[Solution]
(b)(i)














y
x
3
1
2
0
y = f(x)

0 2
y
x


(ii)
















For the two curves to intersect at least 5 points, 4 m> .

2 3


1
1

y
x

Section B: Statistics [60 marks]

6 A school master wants to find out the opinion of students on the variety of food sold at a newly opened
caf in the school. A random sample of 50 students is to be selected from all the 990 students in the
school.

Suggest suitable strata if stratified sampling method is used to select the sample. [1]

A school teacher proposes to hand out questionnaires to the first 50 students buying food from the caf
on a particular day. State a reason why this sampling method will result in obtaining biased
information. [1]

The school master finally decides to collect the information using the school register. She selects at
random a student from the first 20 students in the student register. Next, she selects every 20
th
student
after the first student is chosen in the register.
Explain whether the school master will be able to obtain a random sample of 50 students using this
method. [1]


[Solution]
Suitable strata: race, level, class and/or gender

Why information is biased?
1. Being the first 50 students, they most likely prefer the food at the caf and hence opinions of students
who dont like the food at the caf and do not visit the caf will not be obtained.
2. It is highly possible that the first 50 students consist of students from same class with friends who may
possibly influence each others in their food preference or in answering the questionnaire.


No. Not every student in the population has equal chances of being selected. The 50
th
student could be
selected from the last 10 students and the first 10 students in the register. Thus the first 10 students get a
higher chance of being selected than the rest of the students.

[Note: If do not consider going back to the first 10 students in the register, the school master will only be
able to get up to 49 students and this sample is also not a random sample as the last 10 students are not being
considered at all]


7 The average waiting time for a cab by passengers in the city area is w minutes. In light of the flu
pandemic, the manager of a cab service provider claims that this average waiting time has improved.
A survey was conducted and the waiting times, in minutes, spent by a random sample of 9 passengers
were recorded:

3.5, 2.5, 2.7, 3.2, 2.8, 2.9, 3.1, 3.7, 3.1

Find unbiased estimates of the population mean and variance of the waiting time for a cab in the city.
[2]

Using the above data, a test was carried out at 5% level of significance. Find the range of possible
values of w for the managers claim to be justified. [5]

State any assumption you have made. [1]

[Solution]
Let X denotes the waiting time for a cab in minutes, and
denotes the average waiting time for a cab in minutes.

Using GC,
Unbiased estimate of population mean, 3.0556 3.06 x = = (3sf)
Unbiased estimate of population variance,
2 2
0.38115 0.145 s = = (3sf)

0
H : w =
1
H : w <
Level of significance: 5%

Test Statistics:
8
~
X
T t
s n

=

If
0
H is true,
cal
3.0556
0.38115 9
w
t

=

For one-tailed test at 5% level of significance, the critical value is -1.85955.

To reject H
0
,
cal critical
t t < ,
3.0556
1.85955
0.38115 9
w
<

3.2918 w >
3.30 w > (3sf)


Assume that the waiting time for a cab in the city follows a normal distribution.

8 A 10-digit ternary sequence is a sequence using the digits 0, 1 or 2.
Some examples of such sequences are 0101111001 and 2211001122.
Find the number of possible 10-digit ternary sequences that can be formed when
(i) there is no restriction, [1]
(ii) there are exactly three 0s, [2]
(iii) there are exactly four 0s and five 1s, [2]
(iv) there is at least 1 pair of consecutive digits that are the same. [3]


[Solution]
(i) No. of ways = 3
10
= 59,049


(ii) No. of ways =
7
10
2 15, 360
3
| |
=
|
\ .

[
10
3
| |
|
\ .
ways to place the 3 zeros, followed by 2 choices each for the remaining digits]


(iii) No. of ways =
5
10 6
10!
1 or 1, 260
4 1 4!5!
| | | |
=
| |
\ . \ .



(iv) No. of ways such that there is no pair of consecutive digits that are the same

9
3
2 1, 536
1
| |
= =
|
\ .

[3 choices for the first digit, followed by 2 choices for the 2
nd
digit and so on]

By Complementary Principle,
No. of ways such that there is at least 1 pair of consecutive digits that are the same
= 59,049
9
3
2
1
| |

|
\ .

= 57,513

9 3 women and 4 men take part in the finals of a cooking competition. At the end of the competition, the
judges will pick the top 3 winners to receive the first, second and third prize. Find the probability that
(i) exactly 2 of the winners are men, [2]
(ii) the second prize winner is a man given that exactly 2 men are among the 3 prize winners,
[3]
(iii) the first prize winner is a woman or the third prize winner is a man (or both). [3]


[Solution]
(i) P(exactly 2 of the winners are men)
( ) ( ) ( ) P MMW + P WMM P MWM = +

3 4 3 3!
7 6 5 2!
| |
=
|
\ .
or
3 4
1 2
7
3
C C
C
or
3 4 3 3
7 6 5


18
35
=


(ii) Method 1:
P(2
nd
prize winner is a man | exactly 2 men are winners)

P(2nd prize winner is a man and exactly 2 men are winners )
P(exactly 2 men among the 3 winners)
=

P(MMW) + P(WMM)
18
35
=
| |
|
\ .
[use result in (i)]

4 3 3 3 4 3
7 6 5 7 6 5
18
35
| | | |
+
| |
\ . \ .
=
| |
|
\ .


12
2 35
18 3
35
| |
|
\ .
= =
| |
|
\ .



Method 2: Reduced Sample Space Method
Given that exactly 2 men are winners, the sample space is reduced to the 3 cases:
WMM, MWM and MMW
out of which 2 cases are such that 2
nd
prize winner is a man.
Note that each of the 3 probabilities are the same.
Thus P(2
nd
prize winner is a man | exactly 2 men are winners) =
2
3
.




(iii) Method 1
P(1
st
winner is a woman or 3
rd
winner is a man or both)
= P(1
st
W) + P(3
rd
M) P(1
st
W and 3
rd
M)
3 4 3 4
7 7 7 6
| |
= +
|
\ .


=
3 4 2 5
7 7 7 7
+ =

Method 2
P(1
st
winner is a woman or 3
rd
winner is a man or both)
= 1 P(MMW) P(MWW)

4 3
1
7 6
| |
=
|
\ .
5
7
=

or P(1
st
woman) + P(MMM) + P(MWM)
=
3 4 3
7 7 6
| |
+
|
\ .
=
5
7


Method 3
P(1
st
winner is a woman or 3
rd
winner is a man or both)
= P(1
st
W and 3
rd
not M) + P(3
rd
M and 1
st
W) + P(1
st
W and 3
rd
M)
=
3 2 4 3 3 4
7 6 7 6 7 6
| | | | | |
+ +
| | |
\ . \ . \ .


=
1 2 2 5
7 7 7 7
+ + =




10 In a pilot study done by a private clinic, 8 elderly clients were randomly selected and their Health
Awareness Index were recorded. The table below shows the age (X) of these clients and their Health
Awareness Index (Y). Higher values of Y indicates greater awareness.
Age (X )
51 52 55 59 61 65 70 73
Health Awareness Index (Y ) 70 67 60 53 50 48 47 47
The data set is illustrated in the scatter diagram below:








(i) Calculate the product moment correlation coefficient between X and Y. [2]

(ii) Explain why it is advisable to draw scatter diagram as well as to calculate the product moment
correlation coefficient before interpreting bivariate data. [1]

A proposed model for the above data is ( )
2
69 y p x q = + , where p and q are positive constants
to be determined.

(iii) Obtain the equation of the least squares linear regression line for the proposed model. Calculate
the product moment correlation coefficient between ( )
2
69 X and Y. Determine whether the
proposed model or the least squares linear regression of Y on X is a better model for the data set.
[4]

(iv) Using your regression line obtained in part (iii), calculate the estimated Health Awareness Index
for an eighty year-old man.
Comment on the reliability of your answer. [2]


[Solution]
(i)
, x y
r
= 0.912
(ii) Although
,
0.912
x y
r =
is close to 1, which indicate a strong linear correlation between X and Y, the
scatter diagram shows that the data can be better represented by a non-linear curve.



(iii) Using GC, (X) (Y) (X 69)
2





y
x


The equation of the estimated line of regression of y on (x 69)
2
is:
y = 0.0721(x 69)
2
+ 46.2
2
( 69) , x y
r

= 0.9981
Compare with
x,y
r , 2
( 69) , x y
r

is closer to 1. Hence, y = 0.0721(x 69)
2
+ 46.2 is a better model for
the Data set.

(iv) When x = 80, y = 0.0721(80 69)
2
+ 46.2
= 54.9 (3sf)
As x = 80 is out of the range, this estimate is not reliable.


11 A factory produces a particular brand of oil in bottles of 3 sizes namely Small, Regular and Large. The
amount of oil in each type of bottle, in millilitres, is assumed to be normally distributed as shown in
the table below.

Mean (ml) Variance (ml
2
)
Small 610 25
Regular 1050
2
o
Large 1200 50

(i) Explain why the amount of oil in a Small bottle can be modelled by a normal distribution even
though amount of oil cannot possibly be negative. [1]

(ii) Three Small bottles are randomly chosen. Find the probability that one of the Small bottles
contains more than 615 ml of oil while the other two Small bottles contain less than 602 ml of oil.
[3]

(iii) Find the value of o such that 80% of Regular bottles contained between 1040 ml and 1060 ml of
oil. [3]

(iv) Find the probability that the total amount of oil contained in 2 randomly chosen Large bottles
exceeds 4 times the amount of oil contained in a randomly chosen Small bottle. [3]


[Solution]
Let S, R and L be the amount of oil contained in Small, Regular and Large bottle respectively.
(i) Method 1
Since a normal random variable takes on all real numbers and here P(S < 0) ~ 0 when S is assumed
to be normal, thus S can be modelled by a normal distribution even though S cannot possibly be
negative.

Method 2
The negative values of the amount of oil in a small bottle would more than 3 standard deviation away
from the mean, making the probability of negative values of the amount of oil negligible as we know
that ( ) ( )
P 610 3 5 99.7% X < = .

(ii) Required prob. = ( ) ( )
2 3!
P 615 P 602 0.00143
2!
S S > < = (

(3sf)

(iii) ( ) P 1040 1060 0.8 R < < =

1040 1050
P 0.1 Z
| |
< =
|
\ .
o
or
1060 1050
P 0.1 Z
o
| |
> =
|
\ .



10
1.28155
o

= or
10
1.28155
o
=
7.80 o = (3sf)


(iv) ( ) ( ) ( )
1 2
E 4 2E 4E 40 L L S L S + = =
( ) ( ) ( )
1 2
Var 4 2Var 16Var 500 L L S L S + = + =
( )
1 2
2 ~ N 40, 500 L L S +

( )
1 2
P 4 0 0.0368 L L S + > = (3sf)


12 (a) Along a busy expressway, the number of cars that exceed the speed limit is assumed to follow a
Poisson distribution with a mean of 3.8 cars per day.

Find the probability that in a randomly chosen 3-day period, there are between 9 and 24 cars that
exceed the speed limit. [3]

During a period of n consecutive days, every car driver whose car exceeds the speed limit is each
fined $70. Find the value of n for which the probability of a total fine of $2940 being collected is
the greatest. [3]

(b) At a lucky draw booth it was found from experience that on average, 2 out of 5 people have a
chance to win a prize each day. On a particular day, 50 people are randomly chosen to participate
in the lucky draw. Each person has only one try at the lucky draw. Using a suitable
approximation, find the minimum number of prizes that the organizer of the lucky draw needs to
prepare for each day so that he can be at least 99% sure that he will not run out of prizes on a
particular day. [5]

50 people are randomly chosen to participate in the lucky draw each day.
T denotes the average number of winners per day over a period of 60 days.
State the approximate probability distribution of T and find ( ) P 20.4 T < . [3]


[Solution]
(a) Let X be the number of cars that exceeds speed limit on a 3-day period.
i.e. ( ) ~ P 11.4
o
X
( ) P 9 24 X < <
( ) ( ) P 23 P 9 X X = s s
= 0.701

Let Y be the number of cars that exceeds speed limit in a n-day period.
i.e. ( ) ~ P 3.8
o
Y n ( )
1
poissonpdf 3.8 , 42 Y X =

For ( ) ( ) P 70 2940 P 42 Y Y = = =
to be the greatest,
using GC,
max. probability occurs when n = 11.









(b) Let V be the number of winners (out of 50) on a particular day and r be the number of
prizes he needs to prepare.

( ) ~ B 50, 0.4 V
Since 50 n = is large, 20 5 np = > and ( ) 1 30 5 n p = >

( ) ~ N 20,12 V approx.

( ) P 0.99 V r s >
( ) P 0.5 0.99 V r < + >
0.5 28.0587 r + >
27.559 r > or 28.059 (w/o cc)
Min number of prizes to prepare is 28 or 29 (w/o cc).


( ) E 20 V = , ( ) Var 12 V =
Since n = 60 is large, by Central Limit Theorem,
12
~ N 20,
60
T
| |
|
\ .
i.e ( ) ~ N 20, 0.2 T approx.
( ) P 20.4 0.814 T < ~ (3sf)


End of Paper

Das könnte Ihnen auch gefallen